Download as pdf or txt
Download as pdf or txt
You are on page 1of 92

INTEGRATED REASONING

PRACTICE QUESTIONS

INTEGRATED
REASONING
PRACTICE
QUESTIONS

GMAT
SANDEEP GUPTA
800/800 GMAT,
The Foremost GMAT Trainer in Asia
Founder of Top-One-Percent
Harvard Admit

www.top-one-percent.com
Question1

Outbreak
Genus & Species Illnesses Hospitalizations Deaths
Date
2005-04 E.coli, Shiga toxin-producing 60 11 n/a
2005-05 Salmonella enterica 157 n/a n/a
2005-05 Salmonella enterica 26 11 0
2005-05 Salmonella enterica 7 2 0
2005-06 Listeria monocytogenes 13 13 1
2005-07 Salmonella enterica 52 8 0
2005-07 Salmonella enterica 23 5 0
2005-08 Hepatitis A 39 3 0
2005-09 E.coli, Shiga toxin-producing 34 12 n/a
2005-10 E.coli, Shiga toxin-producing 12 4 0
2005-11 E.coli, Shiga toxin-producing 18 5 0
2005-11 Salmonella enterica 44 2 0
2005-11 Salmonella enterica 84 8 0
2006-05 Vibrio parahaemolyticus 177 2 0
2006-06 Salmonella enterica 115 8 0
2006-06 Salmonella enterica 41 7 0
2006-07 E.coli, Shiga toxin-producing 44 21 n/a
2006-08 E.coli, Shiga toxin-producing 238 103 5
2006-08 Salmonella enterica 715 129 0
2006-09 Clostridium botulinum 4 4 1
2006-09 Salmonella enterica 192 24 0
2006-11 E.coli, Shiga toxin-producing 77 55 0
2006-11 E.coli, Shiga toxin-producing 80 23 0
2006-12 Other chemical 11 1 0

According to the US government's Centers for Disease Control and Prevention (CDC), 24 outbreaks of foodborne
disease occurred across more than one US state in 2005 and 2006. Year and month of the outbreak are indicated.
The etiology (the pathogenic organism, toxin, or chemical that caused the outbreak) is listed under Genus & Species,
since most of the causes are bacteria or viruses. Illnesses, hospitalizations, and deaths caused by the outbreak are
counted, if known. The entry "n/a" means "not available."
For each of the following statements, select Yes if the statement can be shown to be true using the information
provided. Otherwise, select No.
Yes No
There were more than twice as many outbreaks of foodborne illness in the second halves of 2005 and
2006 as in the first halves.

The five outbreaks that resulted in the most individual illnesses caused 5 or fewer deaths.

The percentage of the individual illnesses caused by outbreaks in June that eventually required
hospitalization exceeded 15 percent.
Question 2

The graph shows the change in Variable Cost for 15 branches of a regional department store chain, in thousands of
dollars, over two adjoining periods of time.
From each drop-down menu, select the option that creates the most accurate statement based on the information
provided.
Of the branches that saw an increase in costs during the period from 7/2/2011 to 12/31/2011, the branch with the
median change in cost during the period from 1/1/2011 to 7/2/2011 had approximately a
Select...
net change in Variable Cost over the entire year.

Select...
Approximately of the branches showed a net decrease in costs for the entire period
1/1/2011 to 12/31/2011.
Question 3

Use the drop-down menus to complete each statement according to the information presented in the diagram.
The difference in the number of cellular telephone subscriptions between the United States and Italy in 2008 was
Select...
approximately percent of the difference in the number of subscriptions between China and
the United States in that same year.

Select...
Between 1995 and 2008, the United States experienced approximately a percent increase in
the number of cellular telephone subscribers.
Question 4

Production Fossil fuels Nuclear Other


Year Total Total Crude oil Nat. gas liquids Dry nat. gas Coal Electric Other
1992 70.13 57.66 15.22 2.36 18.38 21.69 6.48 6.00
1993 68.50 55.82 14.49 2.41 18.58 20.34 6.41 6.26
1994 70.89 58.04 14.10 2.39 19.35 22.20 6.69 6.16
1995 71.32 57.54 13.89 2.44 19.08 22.13 7.08 6.70
1996 72.64 58.39 13.72 2.53 19.34 22.79 7.09 7.17
1997 72.63 58.86 13.66 2.50 19.39 23.31 6.60 7.18
1998 73.04 59.31 13.24 2.42 19.61 24.05 7.07 6.66
1999 71.91 57.61 12.45 2.53 19.34 23.30 7.61 6.68
2000 71.49 57.37 12.36 2.61 19.66 22.74 7.86 6.26
2001 71.89 58.54 12.28 2.55 20.17 23.55 8.03 5.32
2002 70.94 56.89 12.16 2.56 19.44 22.73 8.14 5.90
2003 70.26 56.16 12.03 2.35 19.69 22.09 7.96 6.15
2004 70.38 55.91 11.50 2.47 19.09 22.85 8.22 6.25
2005 69.63 55.06 10.96 2.33 18.57 23.19 8.16 6.41
2006 71.04 55.97 10.80 2.36 19.02 23.79 8.21 6.86
2007 71.50 56.25 10.72 2.41 19.62 23.49 8.46 6.80

The table shows US energy production, in quadrillion BTUs, broken down by sources. "Nat. gas" refers to natural gas,
which comes either as liquids or in dry form.
For each of the following statements, select Yes if the statement can be shown to be true using the information
provided. Otherwise, select No.
Yes No

"Other" sources of energy have never accounted for less than 8% of total domestic energy production.

For the years shown, the amount of nuclear electric power produced domestically is negatively
correlated with the year.

Domestic nuclear electric power production increased by more than 10% from 2000 to 2007.
Question 5

The Marsden family is traveling through Europe on vacation and has two days to spend in Barcelona. While planning
the daily activities, each family member has fairly strong preferences. Little brother asked to go to the beach at least
one of the days, but cannot walk for more than 4 hours during a day. Mom loves shopping for local crafts so
requested shopping activities on both days. Big sister will get credit for school so she must visit at least 4 art or
architectural sites during the family’s visit, but Dad said he could not stand to visit more than one art exhibit on a
single day. The family has 12 hours each day to devote to activities, and they have already decided on the following.

Day 1:
Park Güell (Art Exhibit, Bicycle, 3 hours)
Catalunya en Miniatura (Entertainment, Walking, 1 hour)
Olympic Stadium (Sightseeing, Bicycle Tour, 4 hours)
Las Ramblas (Shopping, Bus Tour, 3 hours)

Day 2:
La Segrada Familia (Architecture, Walking, 3 hours)
Barri Gotico (Shopping, Bicycle Tour, 2 hours)
Aeri del Port (Sightseeing, Cable Car, 4 hours)
Nova Icària (Beach, Walking, 1 hour)

Select an activity that can be added to the schedule for the first day. Then select an activity that could be added to
the schedule for the second day. Make only two selections, one in each column.
Additional Day 1 Additional Day 2 Activity

Mirador De Colon (Architecture, Walking, 1 hour)

Poble Espanyol (Shopping, Walking 2 hours)

Montserrat (Sightseeing, Bus Tour, 1 hours)

Museu Picasso (Art Museum, Walking, 2 hours)

Mar Bella (Ocean, Boat Outing, 4 hours)

La Pedrera (Art Museum, Walking, 1 hours)


Question 6-8
Statement by Medical Residency Association president:
Medical residents play an invaluable role in delivering medical care to the citizens of our community, providing round-
the-clock coverage and support to both physician and nursing staff while working to earn our medical licenses.
Residents are responsible for all of the licensed physician tasks, including intake, the ordering of tests and labs,
diagnosis and treatment, including surgery. Nevertheless, we are taken advantage of terribly by the hospitals:
assigned to 24-hour shifts, often undermined or treated poorly by senior doctors – sometimes when patients are
present – and paid well below minimum wage. Our pay is fixed and it is assumed that we work only one-third of the
hours in our assigned shifts; technically, we are considered merely “on call” during these shifts. We are required to be
on the hospital grounds at all times, however, and we work far more than the assumed number of hours per shift.
Over the past month, each resident has averaged 17.5 active hours of work during a 24-hour shift; further, while
working, each resident has averaged only 3.5 hours of sleep at a time. Such working conditions are dangerous for
both patients and residents; some serious mistakes have already been made and were caught only at the last minute
by senior staff.

Statement by Hospital Board spokesperson:


We firmly believe that residents are an integral part of the medical care delivery team at our network of hospitals. At
the same time, medical residencies are a part of the training process to become a physician; until someone has
finished the residency, he or she is not, and cannot reasonably be considered, a licensed physician. Residencies
function in much the manner of an apprenticeship of old: hands-on training under the supervision of a fully trained
mentor. Residents are not expected to work 24 hours in a 24-hour shift, nor anywhere close to that. They are
assigned such shifts merely to ensure that they are on-site when something occurs that would be beneficial to their
training. If residents choose to participate in many routine activities that can ably be handled by the nursing or
medical staff, that is the residents' choice. Further, the entire system is structured to provide both training and
oversight; when a resident is called upon to make a diagnosis or perform a minor procedure, there is always an
experienced physician on hand to verify the diagnosis, take over the procedure, or otherwise correct any potential
errors.

Question 6
For each of the following statements, select Both Agree if it can be inferred that the Medical Residency Association
president and the Hospital Board spokesperson both agree with the statement. If not, select Otherwise.
Both
Otherwise
Agree
The degree to which there is a danger of lasting patient harm caused by medical resident
errors.

Whether medical residents may work more than 8 hours during a 24-hour shift.

The degree to which a resident is as qualified as a licensed physician.

Question 7

For each of the following statements, select Both Accept if, based on the information provided, both the Residency
Association and the Hospital Board would likely accept that the statement is true. Otherwise, select Cannot Infer to
indicate that this likely acceptance cannot properly be inferred.
Both Cannot
Accept Infer
Medical residents should not be expected to work for more than 8 hours during a 24-
hour shift.
A hospital should ensure that medical resident errors are minimized or prevented
entirely.
Medical residents should not be undermined or treated poorly by licensed physicians
when patients are present.
Question 8

If residents are paid $105 per 24-hour shift, what is the difference, rounded to the nearest hundredth of a dollar,
between the hourly wages based upon assumed hours worked and the hourly wage based upon the actual average
hours worked over the past month?
$6.00

$6.49

$6.79

$7.00

$7.13

Question 9

At a certain factory, the number of first-shift workers is 2/3 the number of second-shift workers. Every day, each
second-shift worker can pack 4/3 as many boxes as each individual in the first shift.
In the first column, identify a number that could be the total boxes that the first-shift workers packed on one day; in
the second column, identify the total boxed packed on the same day between the two shifts. Make only two
selections, one in each column.
Total First-Shift Boxes Total Boxes Value

12

24

36
Question 10

The bar graph represents the percentage of the US population, by age, that had health insurance in 1999 and 2008.
The total US population in 1999 (in thousands) was 276,804; in 2008, the total U.S. population (in thousands) was
301,483.

Use the drop-down menus to complete each statement according to the information presented in the diagram.

Assuming independent probabilities in each year, the probability that a U.S. citizen who was 30 in 1999 had health
Select...
insurance both in 1999 and in 2008 is between .

If 35-44 year olds represented 20% of the population in both 1999 and 2008, the number of 35-44 year olds that had
Select...
health insurance in 1999 is approximately than the number of 35-44 year olds who had
health insurance in 2008.

Question 11
Company Service Manager:
Over the past six months, since the first units were sold, we have received fewer customer complaints about
breakdowns of our newly released Dante 5000 stone-cutting machine than we have about any of our other stone-
cutting machines. Therefore, the Dante 5000 must be more reliable than our other stone-cutting machines.
Which of the following, if true would most strengthen the argument above? Which would most weaken it? Make only
two selections, one in each column.
Strengthen Weaken Statement

The Dante 5000 is the most expensive stone-cutting machine produced by the company.

There are other stone-cutting machines that are considered more reliable than the Dante 5000.

The Dante 5000 performed very well in initial testing.

Stone-cutting machines usually break down very quickly under industrial use if they are not
reliable.

Very few customers have purchased a Dante 5000.

The Dante 5000 employs a new technology that is more precise than that used by our previous
stone cutting machines.
Question 12
Train X travels from City A to City B, a distance of 675 miles, in t hours.
Train Y makes the same trip in (t – 3) hours.
Identify speeds in the table that could be the average speeds of train X and train Y, respectively, in miles per hour
(mph). Make only two selections, one in each column.
Train A Train B Speed in mph

45.75

50

56.25

63

67.5

75

Question 13

A cartel is an explicit agreement among competing firms to fix prices, marketing, and production. The goal of a cartel
is to increase individual members' profits by reducing competition. Although firms in many industries have tried to
organize cartels, very few known cartels have lasted for more than 4 or 5 years. The main issue is that the members
of a cartel all have an incentive to cheat and cut prices just a little in order to maximize their individual profits at the
expense of profits of the cartel as a whole.
Indicate the statement in the table that the given information most strongly suggests is true, as well as the statement
that the given information most strongly suggests is false. Make only two selections, one in each column.
True False Statement

No cartel will last more than 100 years.

As long as the members do not cheat, a cartel cannot be broken.

An effective system for preventing members from cheating would not increase the likelihood that a cartel
would survive in the long term.

Private cartels are not legal in most countries.

Cartels are inherently unstable and likely to fail in the long run.

It is extremely difficult for competing firms to agree to fix prices, marketing, and production in the
formation of a cartel.
Question 14

A factory produces one type of widget. This month, the factory raised the price of each widget to X% of the original
price. However, the factory only sold Y% as many widgets as last month, and the total revenue from the sale of
widgets was equal for last month and this month.
In the table, identify the values of X and Y that are consistent with the information provided. Make only two
selections, one in each column.
X Y Value

50

62.5

75

150

160

180

Question 15

The graph shows national debt (the total indebtedness of the government) of several countries, as a percent of the
GDP (Gross Domestic Product, a measure of an economy’s size) of each country. The figures are according to the
International Monetary Fund and refer to the year 2006.

Use the drop-down menus to complete each statement according to the information presented in the diagram.

Select...
Among 15 countries shown in the chart, had the median national debt as a percent of GDP
in 2006.

Among the six ranges of debt levels that are answer choices to this question, the range
Select...
(as a percent of GDP in 2006) contains the most countries.
Question 16

Libraries In collection Lent out to public Users


Year Number Volumes Books AV materials Volumes Books AV materials Registered
2001 508 42,760 37,992 2,422 152,022 137,871 7,205 4,328
2002 483 41,472 36,671 2,438 146,509 132,753 6,976 4,290
2003 473 40,109 35,371 2,475 142,983 129,012 6,941 4,206
2004 467 38,701 34,111 2,456 138,663 124,748 6,683 4,150
2005 351 37,464 33,068 2,383 134,624 121,786 6,059 4,069

The table includes various data about libraries in the Netherlands during the period between 2001 and 2005,
inclusive. Volumes include books, audiovisual (AV) materials, and other materials. "In collection" numbers are
averages (arithmetic means) for the year, whereas "Lent out to public" numbers are totals for the year. All numbers
except for the number of libraries are in thousands.

For each of the following statements, select Yes if the statement can be shown to be true using the information
provided. Otherwise, select No.
Yes No
The year with the fewest "turns" (average number of times an item is lent out to the public) of AV
materials was 2004.
The listed characteristic of Netherlands' libraries that experienced the greatest percent decline from
2001 to 2005 was volumes in collection.

The number of books in Netherlands' libraries' collections per registered user rose from 2002 to 2003.

Question 17
The invented languages of Kurtish and of Laeglish both obey principles of vowel harmony within words, although in
different ways. In both languages, the five vowels (a, e, i, o, and u) are classified either as brutish (a, o, and u) or as
fragile (e and i). In Kurtish, every word can itself be classified as brutish or as fragile, according to the vowels it
contains, if any; there are no mixed-vowel words. In Laeglish, on the other hand, it is possible to have mixed-vowel
words, but within Laeglish words, every consonant (non-vowel letter) or continuous cluster of consonants can only
directly touch vowels of one type or the other.
In the first column, select a word that, according to the constraints given, could be in the Laeglish language, but not
Kurtish. In the second column, select a word that could be in neither language. Make only two selections, one in each
column.
Laeglish only Neither Word

calzral

fjp

aphueitse

brushmen

qudxatroua

hzziigri
Question 18

The graph shows the growth from 1960 through 2008 of nuclear power production in four countries, measured in
millions of tonnes of oil equivalent (toe) annually, according to the International Energy Agency.

Use the drop-down menus to complete each statement according to the information presented in the diagram.

The country with the highest standard deviation in total nuclear power production across all years
Select...
.

In the year 2000, Sweden’s total nuclear power production is equal to approximately
Select...
percent of the total nuclear power production of the Netherlands.
Question 19-21

Country P’s Olympic Committee is determining how much sponsorship money it needs to raise in order to cover its
expenses in helping athletes train for and travel to next summer’s Olympic Games. There are four possible sources of
funding: government grants, income from television rights, team sponsorships, and individual endorsement deals. The
Committee’s task is to determine how to maximize funding from the latter three sources, in order to send as many
athletes as possible to the Games, while minimizing the drain on government funding. Government funding is
provided only when the team is unable to fully fund its activities.
A small number of the highest-profile athletes have secured individual endorsement deals; this money, however,
belongs to that athlete alone and any such athletes are removed from the team funding calculations. An individual
team is considered underfunded if its funding (not including government sources) covers less than 95% of the team’s
expected expenses. Any surplus funding from TV rights or team sponsors reverts to the Olympic Committee and may
be reallocated among other teams. The government will then provide funding to match up to 5% of team expenses or
the amount needed to reach 100% funding, whichever is smaller.
Country P’s athletes are split into 9 primary teams. For each team, the table lists expected funding from TV rights and
team sponsors as well as the percentage of its own expenses that a team is anticipated to be able to cover from TV
rights and team sponsors. In addition, the table indicates the maximum possible level of government funding. All
monetary amounts are in thousands.
Team Percent
Team TV rights Government
Sponsors funded
Basketball 400 250 82 40
Boxing 130 185 92 17
Equestrian 175 415 68 43
Gymnastics 1,100 890 115 —
Soccer 1,800 1,300 126 —
Swimming 770 500 97 39
Tennis 850 330 96 49
Track & Field 1,000 345 103 —
Volleyball 100 130 86 13

Question 19

Approximately how much surplus funding will be available to be shared among the underfunded teams? All answers
are in units of thousands.
940

900

820

640

600
Question 20

For each of the following statements, all of which refer to the 9 teams discussed in the prompt, select Yes if the
statement can be shown to be true using the information provided. Otherwise, select No.
Yes No

More than 50% of the teams are underfunded.

The government will not need to pay the maximum amount of funding that it has made available.

The government funding allocated to the equestrian team is more, as a percentage of total funding
needed by that team, than is the government funding allocated to the basketball team.

Question 21

Assume that TV rights have just been renegotiated and will increase by 15% for each sport. For each of the following
teams, select Yes if the government will still need to provide 5% of the team’s funding. Otherwise, select No.
Yes No

Basketball

Boxing

Volleyball

Question 22

The graph displays textual analysis of four important English-language political declarations from the late 1700's: (A)
the Declaration and Resolves of the First Continental Congress, published in 1774; (B) the Declaration of the Causes
and Necessity of Taking Up Arms, published in 1775; (C) the Declaration of Independence, published in 1776; and (D)
the Declaratory Act, published in 1766.

From each drop-down menu, select the option that creates the most accurate statement based on the information
provided.

Select...
The declaration with the most characters was published in .

The ratio of characters to words in a declaration is most likely to be negatively correlated with
Select...
.
Question 23

Country 1980 1984 1988 1992 1996 2000 2004 2008


Papua New
118.653 118.522 118.947 119.866 120.990 121.791 122.283 122.551
Guinea
Cambodia 119.397 119.373 119.747 120.358 121.131 122.070 122.872 123.244
India 122.071 123.091 124.082 124.653 124.897 125.028 124.815 123.939
Bhutan 122.682 123.305 124.023 124.673 125.245 125.957 126.684 127.180
Thailand 122.792 122.157 121.927 122.400 123.189 123.661 124.169 124.737
Vietnam 123.165 122.870 122.866 123.297 123.931 124.771 125.714 126.579
Nepal 123.824 123.897 123.933 123.951 124.190 124.636 125.104 125.331
Maldives 124.442 124.328 124.807 125.224 125.742 126.516 127.383 127.837
Nigeria 134.494 132.485 130.630 129.751 129.463 130.005 131.302 132.845
Iceland 137.734 134.854 132.709 131.324 130.342 129.931 129.741 129.587
Hungary 138.167 138.904 139.059 138.183 137.064 136.427 136.174 136.057
Austria 138.377 136.506 135.341 134.628 133.740 132.793 131.988 131.360
Netherlands 138.699 136.325 134.666 133.650 132.751 132.138 131.597 131.163
France 138.783 136.678 135.235 134.398 133.617 132.867 131.981 131.046
Serbia 139.020 139.481 139.564 138.925 137.407 136.785 136.946 136.930
Luxembourg 139.621 137.267 135.986 135.363 134.234 133.030 131.866 130.922
Germany 140.020 138.500 137.921 137.906 137.383 136.405 134.670 132.878
Lithuania 140.078 139.683 138.958 137.974 136.550 135.853 136.331 137.282
Niger 140.905 138.788 136.544 135.060 134.677 135.415 137.170 139.417
Greenland 141.265 138.378 136.214 134.682 133.335 132.713 132.748 133.130
Libya 141.705 138.706 135.613 133.067 131.767 132.104 133.919 136.650
Finland 143.124 140.624 138.917 137.483 135.958 134.920 134.824 135.282

The table shows the arithmetic mean Systolic Blood Pressure (SBP) of the male population of selected countries,
measured in mmHg. The means are age-adjusted, as if each country had the same age distribution as the world
population.

For each of the following statements, select Yes if the statement can be shown to be true using the information
provided. Otherwise, select No.
Yes No
The range in blood pressure among nations listed in 1996 was greater than 15% of the blood pressure
of any single nation during 2000.
For the countries/years shown in the table, no nation ranked highest in adult male blood pressure more
frequently than did Finland.
Of the nations listed with a 1980 blood pressure below 125 mmHg, only 4 remained below 125 mmHg
in 2008.
Question 24

350 students at High School High are currently enrolled in French, Spanish, or both French and Spanish. 230 students
are currently enrolled in Spanish.
In the table, identify the number of students currently enrolled in French and the number of students currently
enrolled in both French and Spanish. The two numbers must be consistent with each other and with the constraints
above. Make exactly one choice in each column.
French French and Spanish Number

140

180

200

220

260

280
'

Question 25

A business needs a $240,000 loan for one year and has two options to decide between. Option 1 is a fixed rate loan
for the entire amount, with a simple annual interest rate of 5%. Option 2 involves two loans: a fixed rate loan for
$190,000, with a simple annual interest rate of 4%, and a second fixed rate loan for $50,000, with a simple annual
interest rate of r%. Both options have otherwise identical terms and require payment in full of interest and principal at
the end of the year.
In the first column, identify the interest rate r% at which the two options require the same total interest payment. In
the second column, identify the interest rate r% at which the annual interest on the $190,000 loan in Option 2 would
be exactly twice the annual interest on the $50,000 loan. Make only two selections, one in each column.
Same total interest payments $190,000 loan requires twice the interest Interest rate r%

4.4%

5.2%

6.1%

7.6%

8.8%

9.5%
Question 26

Country Total Deposits Average Age (MY) Total Ore Tonnage (mm) Zinc Grade (%) Lead Grade (%)
Australia 38 1,137 1939.5 6.97 4.39
Canada 72 662 1746.5 5.54 2.44
China 45 622 1698.1 4.63 1.62
Germany 5 275 336.2 3.55 2.00
India 19 1,791 575.3 4.06 1.03
Iran 10 402 470.3 5.37 1.96
Kazakhstan 14 439 551.6 3.40 1.46
Mexico 22 38 456.7 4.09 2.53
Peru 14 15 1104.9 3.86 1.41
Russia 12 551 916.6 3.86 3.01
South
7 1,771 447.7 3.81 1.62
Africa
United
65 346 2464.8 3.88 2.38
States

According to the United States Geological Survey, the 12 countries listed have substantial zinc-lead deposits. The
category total deposits refers to the number of distinct locations where deposits can be found. The average age of the
deposits is given in millions of years (MY). The total ore tonnage of deposits in each country is given in millions of
metric tons (mm). Finally, the zinc grade and lead grade figures represent the percentage of the total ore tonnage
that is made up of the metal in question (zinc or lead).
For each of the following statements, select True if the statement is true based on the information provided;
otherwise, select False.
True False
For the two countries with the greatest total ore tonnage, zinc deposits in tons are more than 1.5
times lead deposits in tons.
Exactly half of the countries with a lead grade percentage of 2% or greater have zinc grade
percentages below the median zinc grade level.

Disregarding units, the range for total ore tonnage is greater than the range for average age.
Question 27

Because of new regulatory requirements in Country X, companies that manufacture non-prescription medications
must disclose all sources of components that are imported from other countries. The new regulations also specify
that, for any individual component, the company must have a minimum of two different suppliers and that, for certain
medications, one country cannot provide more than 70% of a single component used by that manufacturer. Such
regulations are necessary in order to manage contamination risks, reduce the likelihood of financial dependencies on
a single country, and safeguard the uninterrupted supply of critical medicines.
According to the passage, which of the following activities is required to be performed by a non-prescription
medication manufacturer in Country X, and which is NOT permitted? Make only two selections, one in each column.
Required Not Permitted Activities

Buying more than 70% of a component from one country

Importing components from other countries

Reporting to authorities the foreign components used

Using a component for which only one supplier exists outside of Country X

Manufacturing a drug whose components are all from a single supplier


Question 28

The Unified Medical Language System developed by the US National Library of Medicine organizes vast numbers of
medical terms into a semantic network, which is structured somewhat like an organization chart. In the portion
shown, each node (square box) contains a semantic type, to which medical terms are assigned. A solid rectilinear
connector between a higher (“parent”) node and a lower (“child”) node indicates an “is a” relationship: the child node
“is a” specific kind of the parent node. Such relationships are inherited by grandchildren, etc. Dotted curved
connectors with arrowheads indicate other possible associations between nodes, i.e. between some medical terms in
one node and some in another. These associations are described in italics and separately organized with “is a”
relationships, as shown below in a separate hierarchy.

Fill each blank using the drop-down menu to create the most accurate statement on the basis of the information
provided.

Select...
Body Location or Region has an “is a” relationship with .

A node known to be both spatially and functionally related to other nodes is


Select...
.
Question 29
Seven runners, labeled A, B, C, D, E, F, and G, take part in a race. The following is known:
Runners A and E finish consecutively.
Runner B finishes before runner F but after runner A.
Runner D is the fourth to finish the race.
Runner C did not finish first, but did finish within the first 50% of runners.
Based on the information given, identify a runner who could have finished first and a runner who could have finished
fifth. Make only two selections, one in each column.
First Fifth Runner

Question 30-32

Email from account manager to customer


February 17, 9:33 a.m.
I received your order for 50 plastic syringes and 550 single-use needles (one per injection). Also, the portable infusion
pumps are on back-order. I can get you 300 now but the other 900 won’t be available until March. In response to
your question, the plastic syringes typically last for 30 to 40 injections; they are guaranteed to last for a minimum of
20 injections.
Finally, current prices are $20 per box of 50 syringes, $25 per box of 50 needles, and $175 per box of 10 infusion
pump kits.

Email from customer to account manager


February 17, 10:42 a.m.
I noticed that the price of the pumps is set to increase by 15% next month. Will we still receive this month’s price on
the entire order, including the ones we receive next month? My boss is already upset about some of the price
increases earlier this month. Do you offer any large-volume discounts? We’re ordering more than 1,000 pumps.
Hold off on the syringe order; my boss has found syringes that are guaranteed for at least 30 injections. They’re more
expensive ($30 for a box of 20), but he likes the 30-injection guarantee. If we were able to get a volume discount,
though, he might still be willing to give you the order.

Email from account manager to customer


February 17, 1:28 p.m.
Pricing is guaranteed at the time of your order, even if the price increases before you receive the shipment. My
supervisor agreed to a 5% discount for any orders of more than 1,000 units of a single product or a 10% discount on
the whole order if you order a total of 2,000 units for all products combined. I can’t give you a discount on just the 50
syringes, but you already qualify for the 5% discount on the pumps.

Question 30

For each of the following statements, select Yes if the statement can be shown to be true using the information
provided. Otherwise, select No.
Yes No
Because the supplier declined to offer a discount on the syringes, the customer will order syringes from
the other company instead.
The supplier indicates that his syringes typically allow 33 1/3% to 50% more injections than the
guarantee.

The customer is willing to switch to another supplier if her needs are not met satisfactorily.
Question 31

In the exchanges provided, the customer uses all of the following negotiating tactics EXCEPT
demonstrating the value of the existing relationship

asking for a specific consideration

offering a concession that goes beyond what was demanded by the other party

signifying a readiness to walk away from the deal

indicating displeasure with some aspect of the existing relationship

Question 32

For each of the following statements, select Yes if the statement can be shown to be true using the information
provided. Otherwise, select No.
Yes No
On a per-use basis, calculated using the number of guaranteed uses, the syringes guaranteed for 30
injections are less expensive than the syringes guaranteed for 20 injections.
If the customer orders an additional 200 needles, the overall order price will be cheaper than if the
customer only orders 1,200 pumps, 50 syringes, and 550 needles.

The customer is planning to order more than enough syringes for the planned number of needles.
Question 33

Stills Typical product


Officially In continuous Annual capacity Peating level
Distillery Wash Spirit Lomond Style
founded in operation since (thousand liters) (ppm)
Ardbeg 1,815 1,997 1,000 1 1 0 Peated 54.0
Bowmore 1,779 1,779 1,700 2 2 0 Peated 23.0
Bruichladdich 1,881 2,001 1,500 2 2 1 Unpeated 3.5
Bunnahabhain 1,880 1,883 2,500 2 2 0 Unpeated 1.5
Caol Ila 1,846 1,974 5,800 3 3 0 Peated 35.0
Kilchoman 2,005 2,005 115 1 1 0 Peated 25.0
Lagavulin 1,816 1,816 2,200 2 2 0 Peated 35.0
Laphroaig 1,815 1,815 2,200 2 2 0 Peated 40.0

As of January, 2012, eight single-malt whisky distilleries were operating on the island of Islay, off the coast of
Scotland. Some have been in continuous operation since they were officially founded; others have experienced
significant interruptions. Each distillery's annual capacity, in thousands of liters, is listed, together with the number of
stills of each type (wash, spirit, and Lomond) in operation. The typical product of Islay distilleries can be roughly
classified as peated or unpeated. The peating level can be quantified in parts per million (ppm) of phenol absorbed
from peat smoke during the drying of barley malt, an ingredient of the whisky.
For each of the following statements, select Yes if the statement can be shown to be true using the information
provided. Otherwise, select No.
Yes No
The 4 breweries that have been in continuous operation the longest have a higher average (arithmetic
mean) annual capacity than the other breweries.
The median peating level of peated Islay whiskies exceeds that of unpeated Islay whiskies by more
than 35 ppm.
As of 2012, the distillery with the greatest ratio of years since founding to years of continuous operation
to the present is Bruichladdich.
Question 34

A certain species of bacteria grows at a constant rate, its numbers increasing by an unknown multiplier every hour. At
the end of one hour (time t = 1), the bacteria colony’s population size is p; at the end of five hours (t = 5), the
colony’s population has grown to p2.
In the first column, indicate the factor by which the number of bacteria grows each hour. In the second column,
indicate the population at the start of the time period described (at time t = 0). Make only two selections, one in each
column.
Growth Factor Population at t = 0

p1/4

p1/3

p1/2

p3/4

p3/2
Question 35

The diagram displays the number of earthquakes that took place around the world on 6 successive days, as recorded
by the US Geological Survey. The magnitude of the earthquakes ranges from 1 to 6 on the Richter scale, rounded
down.

Use the drop-down menus to complete each statement according to the information presented in the diagram.

Excluding February 14, the day on which occurred the greatest proportion of earthquakes of magnitude of 3 or
Select...
greater is .

Select...
Of the earthquakes shown, approximately occurred on February 14.
Question 36

The chart above displays Forbes data for the distribution of billionaires by country in 2007, when there were 946
billionaires worldwide.
Use the drop-down menus to complete each statement according to the information presented in the chart.

Select...
In 2007, the United States had approximately more billionaires than Russia, the United
Kingdom, and Germany combined.

If the portion labeled “Other” on the chart above represented 7 countries with an equal number of billionaires, then
Select...
each of these countries would have approximately billionaires.
Question 37

Size of tax Average income per tax household % % dividends &


State Population
household ($) wages interest
Colorado 5,029,196 2.39 60,834 72.0% 5.2%
Connecticut 3,574,097 2.24 77,059 72.3% 6.1%
Georgia 9,687,653 2.44 50,320 76.0% 4.5%
Idaho 1,567,582 2.48 47,999 69.6% 5.3%
Maryland 5,773,552 2.27 66,206 75.1% 4.6%
New
2,059,179 2.38 44,129 70.7% 4.9%
Mexico
Texas 25,145,561 2.58 51,747 76.8% 4.4%
Vermont 625,741 2.07 50,394 69.1% 6.6%
Wyoming 563,626 2.30 62,077 64.6% 6.3%

The table contains information about 9 US states. Population figures are from the 2010 census. Size of tax household
is computed by dividing the population by the total number of tax returns filed in 2007 to the Internal Revenue
Service (IRS), assuming that each return corresponds to a tax household. The remaining numbers are also 2007 IRS
figures. Average income per tax household is an arithmetic mean in 2007 US dollars. “% wages” represents the
portion of income derived from wages and salaries, while “% dividends & interest” represents the portion of income
derived from dividends and interest.
Each column of the table can be sorted in ascending order by clicking on the drop-down menu above the table and
choosing the heading of the column by which you want the table to be sorted.
For each of the following statements, select Yes if the statement can be shown to be true using the information
provided. Otherwise, select No.
Yes No

The percent of income derived from wages and salaries is negatively correlated with population.

The state with the lowest percent of income per tax household derived from sources other than wages,
salaries, dividends, and interest has an income per capita of more than $20,000.
Per household, the average (arithmetic mean) income in dollars from dividends and interest is less in
Georgia than in New Mexico.
Question 38-40

Email from executive director to charity staff


May 5, 3:14 p.m.
Our annual Grand Ball and Silent Auction will be on October 12th this year. The venue has a maximum capacity of
1,098 people (including servers and other workers). We need to determine how to maximize attendance while not
going over the capacity limit.
In addition, I’d like to offer 100 lots for auction, and we will be running a pledge table again this year. Our goal is to
raise a total of $250,000 from the auction and pledges combined.

Email from auction coordinator in response to the director’s May 5, 3:14 p.m. message
May 6, 9:33 a.m.
Last year, we invited 1,813 people and 1,492 accepted. Of those, 1,378 actually attended. We could probably plan on
similar response and attendance rates this year. All of the companies who gave items to be auctioned last year are
donating again this year, except for 2, so that accounts for 56 lots. In addition, my team has already secured an
additional 12 items for the auction; I’m certain we can reach 100 items. Last year, we raised $131,000 from pledges
made during the event and the auction brought in an additional $73,000. This year’s lots should go for about the
same price on average, so the 42 additional auction lots this year should be enough to raise the additional $50,000
needed.

Email from executive director to auction coordinator in response to the auction coordinator’s May 6, 9:33 a.m.
message
May 9, 12:48 p.m.
Maybe we should think about moving to a larger venue so we make sure that we don’t exceed the capacity limit. Last
Saturday night, the opera’s benefit gala was almost shut down because they had too many people. Assume that
about 10% of the people will be servers and other workers. We would lose our deposit though. Well, if our projections
are accurate, we should be fine at this venue.

Question 38
If the charity is able to raise an average of $1,500 per lot (auctioned item), and assuming that the maximum capacity
of the venue will be met, what must be the approximate average pledge per invited guest in order to reach the
$250,000 goal?
$90

$100

$110

$120

$130

Question 39

Consider each of the following statements. Indicate Yes if the information in the three emails support the statement;
otherwise, indicate No.
Yes No
The executive director would rather change to a larger venue than risk exceeding the capacity of the
currently-booked venue.
The auction coordinator believes that the charity will be able to reach its goal to raise $250,000 at this
year’s event.
The auction coordinator is certain that the charity has the necessary data to determine how many
people to invite in order to maximize attendance while not exceeding the capacity limit.
Question 40

For each of the following plans, select Yes if the plan will definitely allow the charity to reach its planned goal to raise
$250,000 from the auction and pledges. Otherwise, select No.
Yes No
The charity follows its plan as stated in the emails, maximizing the number of invited guests and
achieving the same rates as in the prior year for the pledge money raised per person and the auction
money raised per item auctioned.
The charity moves to a larger venue; 1,500 invited guests attend but the charity is able to offer only 90
items for auction.
The charity maximizes its number of invited guests at the planned venue; it auctions 100 items at an
average of $1,300 per item and receives an average of $150 per invited guest in pledges.

Question 41

The Standard Model of Particle Physics

Each circle in the diagram represents a type of subatomic particle in the Standard Model, according to which particles
fall into two classes, fermions and bosons. Fermions are either leptons or quarks. Bosons carry particular forces
between other particles. Particle types linked directly by a blue line interact by means of the force associated with the
boson type; particle types not so linked do not interact. Photons carry the electromagnetic force; W and Z bosons
carry the weak nuclear force; and gluons carry the strong nuclear force. Finally, particles that interact with the still-
hypothetical Higgs boson obtain their mass via this interaction, although even massless particles (such as photons)
are subject to gravitational effects via general relativity.

From each drop-down menu, select the option that creates the most accurate statement based on the information
provided.
Select...
The force that affects quarks but not leptons is the force.

Select...
A boson that cannot carry a force between other bosons of precisely the same type is a .
Question 42

Colleague A: “The office policy to limit employee parties for personal events such as birthdays, marriages, and
retirement to one per month is counterproductive. No one employee feels personal attention because his or her
special event is lumped in with so many others.”

Colleague B: “Individual parties would likely increase the costs, borne by the participating employees, of refreshments
and gifts. Productivity might suffer and there would be a higher chance that a particular milestone might be
overlooked.”

In the first column, indicate a statement to which Colleague A would likely object. In the second column, indicate a
statement with which Colleague B would likely respond to Colleague A’s objection. Select only two statements, one
per column.

Colleague A’s Colleague B’s


objection response
here are significant disadvantages involved with changing the current policy
regarding office parties.
There are significant advantages involved with changing the current policy
regarding office parties.
The company should maintain its current policy regarding office celebrations
for personal events.
The company should offer to defray the costs of employee parties for
personal events.
The company should continue to let employees cover the costs of parties for
personal events.
Question 43

Description Gene ID Serpin Clade Act / Hyp Length (aa) Max Identity (%)
Alpha-1-antiproteinase
423,434 A Hyp 425 29
2
Alpha-1-antitrypsin 423,435 A Hyp 437 32
Angiotensinogen 421,543 A Hyp 476 24
Heparin cofactor 2 395,877 D Hyp 488 23
Neuroserpin precursor 425,002 I Act 410 35
Ovalbumin precursor 396,058 B Act 386 64
Ovalbumin-related
420,898 B Hyp 397 99
protein Y
Plasma protease C1
423,132 G Hyp 503 27
inhibitor-like
Protein Z-dependent
423,432 A Hyp 439 31
protease inhibitor
Serpin B6 420,895 B Act 379 44
Serpin H1 precursor 396,228 H Act 405 33

The table shows results of a search query to locate regions of similarity in genetic sequences on the Gallus
gallus (chicken) genome. When a search was performed with a sample protein sequence, the database returned 11
protein sequence matches of varying quality. The gene ID reflects the sequence’s position on the genome. Each
protein belongs to one serpin clade, or category. Genes that have already been shown to encode proteins are
identified as “Act”; those that have not yet been proven to do so are labeled “Hyp.” The length of the sequence is
shown measured in amino acids (aa). The maximum identity percentage reflects the degree to which the protein
sequence matches the initial sample used for the search.
Each column of the table can be sorted in ascending order by clicking on the drop-down menu above the table and
choosing the heading of the column by which you want the table to be sorted.

For each of the following statements, select Yes if the statement is true based on the information provided; otherwise,
select No.
Yes No

For each of the Hyp sequences listed, the maximum identity is less than 50%.

Sequences in serpin clade A have a lower average (arithmetic mean) length than sequences in serpin
clade B.
When choosing one of the 11 sequences at random, the probability that the length will be 415 or longer
is greater than ½.
Question 44
Students in the tenth grade at a certain high school must take at least one science course: chemistry, physics, or
biology. Each of these classes contains 20 students, and any two classes have the same number of students in
common. Five students are taking all three classes simultaneously.
Using the information given, identify a possible number of students in the tenth grade taking only one science class
and the corresponding number of students common to any pair of classes. The numbers must be consistent with each
other. Make only two selections, one in each column.
Only One Science Common to A Pair of Classes Number

24

33

39

Question 45

The diagram above displays the proportions of households using various residential heating sources in the entire
United States and in only the Midwest region (within the US) as recorded by the EIA Residential Energy Consumption
Survey (RECS). Included in the survey are 113.6 million US households, of which 25.9 million are located in the 12
Midwestern states (IL, IN, IA, KS, MI, MN, MO, NE, ND, OH, SD, and WI).

Fill each blank using the drop-down menu to create the most accurate statement on the basis of the information
provided.

The number of households in the Midwest that do not use electricity for heating is approximately
Select...
of the number of households in the US that use electricity.

In the Midwest region, the average number of households using either propane/LPG or natural gas for heating is
Select...
approximately thousand per state.
Question 46

Scientist: Due to their simple fabrication process, low production cost, and high efficiency, dye-sensitized solar cells
(DSSCs) are provoking significant interest in the field of renewable energy sources. As a result, a variety of
investigations have been made into how DSSCs could be made more efficient, because the technology has the
potential to be manufactured economically enough to compete with other solar cell technologies. Titanium dioxide
nanoparticles have been widely used as working electrodes for DSSCs, because they provide higher efficiency than
any other metal oxide semiconductor investigated; the highest conversion efficiency under air mass irradiation
reported for these devices to date is about 11%. Considering this initial success, DSSCs are going to be the most
useful form of solar cell technology.
Indicate two different statements as follows: one statement identifies an assumption required by the scientist’s
argument, while the other identifies a possible fact that, if true, would provide the most support for the required
assumption. Make only one selection in each column.
Assumption Possible
required fact
Of the six most viable forms of solar cell technologies, DSSCs are the least expensive
and most efficient.
Technological advancements are certain to bring the cost of DSSCs down to at least
25% of current costs.

DSSCs are considered a plausible solution for the current energy crisis.

No other nanoparticles are as efficient as titanium dioxide.

A conversion efficiency under air mass irradiation of 11% is considered above


average.
There are no other forms of solar cell technology that will be more useful than
DSSCs.
Question 47

The graph displays the average (arithmetic mean) fourth grade scores for several countries or regions on the Trends
in International Mathematics and Science Study (TIMSS) mathematical assessment test.

Use the drop-down menus to complete each statement according to the information presented in the diagram.

New Zealand had a fourth grade TIMSS score that was both higher in 2003 and lower in 2007 than
Select...

Select...
had an approximate change in fourth grade TIMSS score of 9% between the
years 1995 and 2007.

Question 48

Car A and Car B are driving with constant, but different, speeds in the same direction. Initially, Car A lags behind Car
B by 20 miles, but after 4 hours Car A is 4 miles ahead.
Using the information given, identify a number that could have been the speed of Car A and the speed of Car B, both
in miles per hour (mph). Make only two selections, one in each column.
Car A’s Speed Car B’s Speed Value (mph)

44

48

49

54

57

59
Question 49

This diagram displays the percent of population that is obese versus the number of active farmers markets for 35
states and territories in the US. The size of the bubble gives a visual representation of the population of each state or
territory. The smallest bubble represents a population of approximately 600,000, while the largest represents a
population of approximately 13 million.

Fill each blank using the drop-down menu to create the most accurate statement on the basis of the information
provided.

Under the simplification that the 5 smallest states all have about the same population, together they must have
Select...
approximately farmers markets per 100,000 people.

Select...
The ratio of obese people to farmers markets in the two largest states is closest to .
Question 50

Film Production Speech Technical


Country
Title Year No. Language(s) No. Length B/W? Medium
/ies
The Basque Ball: Skin Against
2004 Spain 1 Spanish, Basque 2 106 No 35mm
Stone
The Big Durian 2004 Malaysia 1 English 1 75 Yes HD
The Corporation 2004 Canada 1 English 1 145 Yes 35mm
Disbelief 2004 USA, Russia 2 English, Russian 2 105 No 35mm
Garden 2004 Israel 1 Hebrew, Arabic 2 85 No 35mm
Investigation into the Invisible
2004 France 1 Icelandic 1 90 No 35mm
World
Journeyings and Conversations 2004 India 1 Hindi 1 88 No 16mm
Repatriation 2004 South Korea 1 Korean 1 149 Yes HD
Screaming Men 2004 Finland, Denmark 2 Finnish, English 2 76 No 35mm
Russian, Chechen, Arabic,
The 3 Rooms of Melancholia 2005 Finland 1 4 106 Yes 35mm
Finnish
Dhakiyarr vs. the King 2005 Australia 1 English, Yolngu Matha 2 56 Yes HD
Grizzly Man 2005 USA, Canada 2 English 1 100 No HD
I Am Cuba, the Siberian
2005 Brazil 1 Spanish, Russian, Portuguese 3 90 Yes 35mm
Mammoth
Spain, Nicaragua,
El Inmortal 2005 3 Spanish 1 80 No 35mm
Mexico
The Liberace of Baghdad 2005 United Kingdom 1 English 1 75 No HD
Russian, Yiddish, English,
Odessa Odessa 2005 France, Israel 2 4 96 No 35mm
Hebrew
Shake Hands with the Devil: ...
2005 Canada 1 English, French 2 91 No HD
Romeo Dallaire
Shape of the Moon 2005 Netherlands 1 Bahasa Indonesian, Javanese 2 92 No 35mm
Unknown White Male 2005 United Kingdom 1 English 1 80 No HD
Wall 2005 France, Israel 2 Hebrew, Arabic 2 95 No 35mm
Yang Ban Xi: The 8 Model Works 2005 Netherlands 1 Mandarin 1 90 No HD

Twenty-one films participated in the World Documentary Competition at the Sundance Film Festival in 2004 and 2005.
The country or countries of production, as well as the languages spoken in each film, are listed. The length of each
film is given in minutes. Each film was shot either in color or in both color and black & white (b/w); moreover, the
shooting medium was either 35 millimeter film (35mm), 16 millimeter film (16mm), or Sony HD Cam.
For each of the following statements, select 2004 if the statement is true of the movies produced in 2004.
Select 2005 if the statement is true of the movies produced in 2005. The comparison in the statement is always with
the other, unchosen year.
2004 2005

A higher percentage of the movies produced in this year had English as one of the spoken languages.

The movies from this year that were shot entirely in color have a longer average length (in minutes).

A higher percentage of movies that were shot in 35 mm were also shot in b/w in this year.
Question 51-53

Transuranic Actinides

The periodic table of chemical elements contains 11 transuranic actinide elements, with atomic numbers ranging from
93 to 103. An element is defined by its atomic number, the number of protons in the atomic nucleus. Nuclei with the
same atomic number but different numbers of neutrons are isotopes of the same element. The number of possible
isotopes that have been attested in the laboratory is listed in the table shown for each element. The mass number of
an isotope is the total number of protons and neutrons. All of the transuranic actinides are unstable. For any element,
the most stable isotope is the one with the longest half-life, or time until half of the original amount present decays.
The possible decay modes of the most stable isotope include Alpha particle emission, which drops the atomic number
by 2 and the mass number by 4; Spontaneous Fission (SF), in which the nucleus breaks into several fragments; Beta
particle emission, which increases the atomic number by 1 but leaves the mass number unchanged; and Electron
Capture (EC), which decreases the atomic number by 1 but leaves the mass number unchanged.

Elements
Mass # Smallest Density
Melting
of Most Mass # (kg per
Atomic Point (in
Element Stable of an cubic
Number Kelvins) of
Isotope Attested meter) of
MSI
(MSI) Isotope MSI
93 Neptunium 237 225 910 20,450
94 Plutonium 244 228 913 19,816
95 Americium 243 231 1,449 n/a
96 Curium 247 232 1,613 13,510
97 Berkelium 247 235 1,259 14,780
98 Californium 251 237 1,173 15,100
99 Einsteinium 252 240 1,133 13,500
100 Fermium 257 242 1,800 n/a
101 Mendelevium 258 245 1,100 n/a
102 Nobelium 259 249 1,100 n/a
103 Lawrencium 262 251 1,900 n/a

Decay info
Atomic Time Unit Decay
Half-life
Number of Half-life Modes
93 2,140,000 years Alpha; SF

94 82,000,000 years Alpha; SF

95 7,370 years Alpha; SF

96 15,600,000 years Alpha

97 1,400 years Alpha

98 898 years Alpha; SF


Alpha;
99 1.29 years
Beta; EC
100 100.5 days Alpha; SF

101 51.5 days EC


Alpha; SF;
102 58 minutes
EC
103 3.6 hours SF; EC
Question 51

For each of the following statements, select Yes if the statement can be shown to be true using the information
provided. Otherwise, select No.
Yes No
The four transuranic actinide elements whose most stable isotopes have the shortest half-lives all
permit electron capture as a decay mode.

The most stable isotope of californium can decay directly to the most stable isotope of curium.

The difference between the highest melting point and the median melting point (as a percent of the
highest melting point) is greater than the difference between the highest density and the median
density (as a percent of the median density).

Question 52

For each of the following statements, select Can Be Determined if the statement can be determined with the
information given. Otherwise, select Cannot Be Determined.
Can Be Cannot Be
Determined Determined
The atomic number of an atom that was once neptunium but that has
decayed through one half-life cycle
The atomic number of an atom that was once mendelevium but that has
decayed through one half-life cycle
The atomic number of an atom that was once einsteinium but that has
decayed through one half-life cycle

Question 53

Of the elements with two possible decay modes, which represents the number of protons in the element with the
largest difference between the number of neutrons in its most stable isotope and the number in its smallest attested
isotope?
93

94

95

100

103
Question 54

A city is hosting a Swiss-system chess tournament called Chessmaster. In a Swiss-system tournament, every player
plays every other player, and no one is eliminated. At Chessmaster, each player who wins a match receives two
points; those who draw a match each receive half a point; those who lose a match each lose one point; and every
player can choose to sit out one match (and neither receive nor lose points). Player X has 4 points after 6 matches.
From the available options, select a number of wins and a number of combined draws and losses that would result in
Player X’s score. The answers must be jointly consistent with the outcome. Make only one choice in each column.

Wins Draws + Losses Number

5
Question 55

A natural history museum is planning two butterfly exhibits. Each exhibit has a theme. The theme of the first exhibit
is “Butterflies of Asia,” and the theme of the second exhibit is “Blue Butterflies.” In order to support the themes, at
least 3 of the 4 butterflies in the “Butterflies of Asia” exhibit must exist in Asia and at least 3 of the 4 butterflies in the
“Blue Butterflies” exhibit must be at least partially blue or purple colored. Three butterflies have already been
selected for each exhibit:

“Butterflies of Asia”

Common name Coloration Natural habitat


Philippines, India, Nepal, China, and
Red Lacewing red and black
Indonesia
Leopard Lacewing gold, brown, and white India, China, Malaysia and Singapore
Australian Painted Lady orange and brown Australia and New Zealand

“Blue Butterflies”

Common name Coloration Natural habitat


Great Nawab white, black, and orange Asia
Blue Morpho blue Central and South America, also Mexico
Great Spangled light brown and light blue North America

From the list below, select one butterfly that could be added to either exhibit and one that could be added to neither
exhibit. Make only two selections, one in each column.
Either exhibit Neither exhibit Butterfly
Name: Peacock Butterfly
Coloration: rusty red
Natural habitat: Europe and parts of Asia
Name: Great Nawab
Coloration: white, black, and orange
Natural habitat: Asia
Name: Sapho Longwing
Coloration: blue, white, and black
Natural habitat: Costa Rica
Name: Monarch Butterfly
Coloration: orange and black
Natural habitat: North America
Name: Pipevine Swallowtail
Coloration: light blue and gray
Natural habitat: United States and Mexico
Name: Emerald Swallowtail
Coloration: blue, green
Natural habitat: Pakistan, India, South Asia
Question 56

A sports coach intends to choose a team of players from a pool of candidates. The coach wants to be able to have
more than 20 but fewer than 25 distinct possibilities for the composition of the chosen team, with at least as many
candidates chosen for the team as those not chosen.
Identify the number of candidates in the pool and the number of players on the team that are consistent with the
coach’s intentions. Make only two selections, one in each column.
Candidates in Pool Players on Team Number

Question 57

From each drop-down menu, select the option that creates the most accurate statement based on the information
provided.

Select...
The bedrooms represent approximately of the apartment’s indoor square footage.

Ignoring the outdoor patio, the approximate distance from one corner of the apartment to the farthest corner from it
Select...
is closest to feet.
Question 58

Heavy traffic on Austia’s freeways often results in much longer than expected commute times. Home construction in
outlying “commuter” neighborhoods is expected to continue to increase steadily and the total traffic on the main
freeways is projected to increase by 25% over the next 15 years. The Austia Regional Mobility Authority plans to
increase road capacity by only 10% over this same period. Austia Regional Mobility officials predict that this increase
is sufficient to ensure that commute times will not increase.

Which of the following, if true, provides the strongest grounds for the officials’ prediction? Which would cast the most
doubt on it? Make only two selections, one in each column.

Strengthen Weaken Statement


The Austia Regional Mobility Authority is also planning a 5% increase in capacity for
downtown streets.
Most of the growth in the new suburbs is on the north side of the city, where most
of the new residents will work, primarily using Austia’s freeways only to travel to
central Austia during off-peak hours.
Since Austia has limited public transportation outside of the city core, commuters
who drive from the outlying neighborhoods to the city center to work have no
practical alternative to driving available.
The projected increase in highway utilization is expected to occur gradually over
time.
By converting wider-than-typical shoulders into traffic lanes, the Austia Regional
Mobility Authority expects to able to increase capacity on some local roads with only
minor amounts of construction.
In part due to the increased tax revenue due to the new residents, the Austia
Regional Mobility Authority believes that it will be able to increase freeway capacity
without raising the local tax rate.

The officials predict that the 10% increase is freeway capacity will be sufficient to ensure that commute times will not
increase, even with the expected 25% increase in total traffic.

This plan assumes that Austia’s traffic will not increase by 25% with the 25% increase in population. In order for a
10% increase in capacity to be enough for a 25% increase in population, either the new population must not use the
Austia freeways as much as the old population does, or there must be some unused capacity on the existing
freeways.
Question 59

Vessel Charter
Name Class Year DWT OpExp Type Duration Expiry date Daily rate
Agamemnon
IMO II/III C/P 2008 51,238 6,500 TC 3 2007-11 22,000
II
Agisilaos Ice Class IA 2006 36,760 5,500 TC 1 2007-01 11,850
Aiolos Ice Class IA 2007 36,725 250 BC 8 2011-01 15,000
Akeraios Ice Class IA 2007 47,781 5,500 TC 1 2007-04 12,500
Aktoras Ice Class IA 2006 36,759 250 BC 8 2010-05 15,000
Alexandros
IMO II/III C/P 2008 51,258 250 BC 10 2013-11 13,000
II
Alkiviadis Ice Class IA 2006 36,721 7,000 TC 2 2008-05 12,838
Amore Mio
Crude Oil 2001 159,982 8,500 TC 1 2007-11 25,000
II
Anemos I Ice Class IA 2007 47,782 5,500 TC 3 2009-07 14,259
Apostolos Ice Class IA 2007 47,782 5,500 TC 2 2008-08 14,000
Arionas Ice Class IA 2006 36,725 5,500 TC 1 2007-08 11,850
Aris II IMO II/III C/P 2008 51,218 250 BC 10 2014-06 13,000
Aristofanis Chem/Prod 2005 12,000 5,500 Spot - - -
Aristotelis II IMO II/III C/P 2008 51,226 250 BC 10 2014-04 13,000
Assos Ice Class IA 2006 47,872 500 BC 5 2010-02 16,825
Atlantas Ice Class IA 2006 36,760 250 BC 8 2010-02 15,000
Atrotos Ice Class IA 2007 47,786 500 BC 5 2010-02 16,825
Attikos Chem/Prod 2005 12,000 5,500 Spot - - -
Avax Ice Class IA 2007 47,834 5,500 TC 1 2007-03 12,500
Axios Ice Class IA 2007 47,872 5,500 TC 1 2007-01 12,591
Ayrton II IMO II/III C/P 2009 51,260 6,500 TC 2 2008-02 22,000

An international tanker company recently released data on the 21 vessels of four different classes in its fleet, including
name, class, year built, deadweight tonnage (DWT, the weight a ship can safely carry in metric tons), and daily
operating expense (OpExp) incurred by the company in US dollars. 19 vessels were under medium- to long-term
leases, either time charters (TC) or bareboat charters (BC), with an average remaining term of 4.6 years as of
January 31, 2011. Under a time charter, the vessel's owner provides crewing and other operational services; under a
bareboat charter, the charterer must provide such services. The other 2 vessels were trading on the spot market for
short-term charters. Expiry date (year-month) indicates the earliest possible redelivery date at the conclusion of the
charter. The daily charter rate (net) is the payment, after commissions, from the charterer to the owner, in US dollars.
For each of the following statements, select Yes if the statement can be shown to be true with the information
provided. Otherwise, select No.
Yes No
Every charterer of a TC vessel capable of transporting more than 50,000 metric tons of goods is paying
less than $800,000 a month, net, to the owner. (Assume a month has 30 days.)

The median daily rate of a BC vessel is greater than the median daily rate of a TC vessel.

For all Ice Class IA vessels, the daily operating expense never exceeds 55% of the daily rate.
Question 60

During labor, electronic fetal monitors measure both fetal heart rate (FHR) in beats per minute (bpm) and the
strength of uterine contractions in millimeters of mercury (mmHg). Measured by ultrasound, FHR is tracked on the top
graph, while uterine activity (measured by a tocotransducer) is simultaneously tracked on the bottom graph. The
same moment in time is represented by the same horizontal position on both graphs; the smallest horizontal division
represents 10 seconds. Baseline FHR is the average FHR, rounded to the nearest 5 bpm, over a 10-minute window
that excludes periods of uterine activity, FHR accelerations and decelerations, and periods of marked (>25 bpm) FHR
variability. The ultrasound displays a "late" deceleration between minutes 3 and 5. The duration of a uterine
contraction is measured from the onset of a sharp increase in uterine pressure from a baseline (typically 0 mmHg as
shown) to the return to the baseline.

Fill each blank using the drop-down menu to create the most accurate statement on the basis of the information
provided.

On the basis of the 6 minutes of data shown only, the baseline FHR can be estimated most nearly to be
Select...
bpm.

Select...
The uterine contraction shown lasts approximately seconds.
Question 61

The four categories of milk shown in the graph above represent 100% of total milk consumption in the country of
Malactica for the years listed.

Use the drop-down menus to complete each statement according to the information presented in the diagram.

The type of beverage milk that experienced the greatest magnitude percent change in consumption in Malactica from
Select...
1980 to 2005 was .

Plain reduced-fat milk consumption as a percentage of total beverage milk consumption was lowest in
Select...
.

Question 62

Earthquakes occur when energy in the earth’s crust is suddenly released, forcing tectonic plates to shift. Earthquakes
are classified as foreshocks, main shocks, or aftershocks. What differentiates one category from another is their
relation to each other in space and time. A foreshock is only a foreshock if it occurs before a bigger quake on the
same fault system. Similarly, an aftershock occurs only after a bigger quake on the same fault system. Interestingly,
the probability that an earthquake will trigger a bigger earthquake does not depend on the magnitude of the first
earthquake, but rather is related to the location of that first quake and its interaction with the fault system.
In the first column, indicate the statement that the given information most strongly suggests is true. In the second
column, indicate the statement that the given information most strongly suggests is false. Make only two selections,
one in each column.
True False Statement

All main shocks are preceded by foreshocks.

All main shocks are followed by aftershocks.

Aftershocks are more common that foreshocks.

Foreshocks are generally weaker than aftershocks.

Main shocks can be triggered by foreshocks.

An aftershock can be a bigger earthquake than the main shock that preceded it.
Question 63

Use the drop-down menus to complete each statement according to the information presented in the diagram.

If, in 2007, the populations of Minnesota (MN) and of Iowa (IA) were 5.3 million and 3.0 million, respectively, then
the maximum amount that Minnesota spent on Public Elementary and Secondary School could have been
Select...
greater than the amount that Iowa spent on Public Elementary and Secondary School in 2007.

If, theoretically in 2007, the population of Missouri (MO) was 5.9 million and the total Public Elementary and
Secondary School expenditures for Missouri (MO) and Kansas (KS) were equal, then the population of Kansas (KS)
Select...
could have been .
Question 64

Apartment Monthly Rent ($) Square Feet Rent per Square foot ($) Parking Pool Fitness Center Washer and Dryer
A 1,750 626 2.80 Covered Yes Yes No
B 1,500 778 1.93 None Yes No No
C 1,400 660 2.12 None No Yes No
D 1,880 994 1.89 Lot Yes Yes Yes
E 1,220 678 1.80 Lot Yes Yes No
F 950 712 1.33 None No No No
G 1,530 620 2.47 Garage Yes No No
H 1,125 559 2.01 None Yes Yes Yes
I 1,575 680 2.31 Garage Yes Yes Yes
J 1,350 612 2.21 Covered Yes Yes No
average 1,428 692 2.00 - - - -

The table shows all of the apartments with 1 bedroom and 1 bathroom that are available within 1 mile of business
school B. The "average" row contains arithmetic means.

For each of the following statements, select Yes if the statement can be shown to be true using the information
provided. Otherwise, select No.
Yes No

All of apartments in the table that have a washer and dryer also have a pool.

An apartment with the median square footage of the apartments in this table would be slightly larger
than an apartment with the average (arithmetic mean) square footage of the apartments in this table.
There is a positive correlation between rent per square foot and the state of having either covered or
garage parking.
Question 65

A certain game features 2 buckets: one with large marbles numbered 1 through 50, inclusive, and another with small
marbles numbered 1 through 10, inclusive. To play the game, a person chooses 3 large marbles and 1 small marble.
During one game, Bobby chose one marble with the number 48 on it. The average of the numbers on all the marbles
he chose in that game, both large and small, was 18.
In the first column, identify a number that could have been the sum of the other two large marbles chosen. In the
second column, identify the corresponding value of the small marble chosen. The two choices must be consistent with
each other. Make only two selections, one in each column.
Sum of Large Small Value

10

11

13

19

Question 66

Six children – A, B, C, D, E, and F – and four parents – W, X, Y, and Z – are split into three groups for a camping trip.
The following rules must hold:
Every group must have at least one parent.
E must be in the same group as F.
Y is not in the same group as D or A.
If F is in group 2, then A is in group 1.
No group has more than two children.
A and B must be split among groups 1 and 3.
Based on the information given, identify a parent that could be in group 1 and a child that could be in group 3. Make
exactly one selection in each column.
Parent in Group 1 Child in Group 3 Person

Z
Question 67 to 69

Review Board
Monkton University has established a board which reviews all applications for experiments to be conducted by
students or faculty. Researchers submit proposals to one or more committees, each of which is responsible for certain
kinds of research subjects, and the committees’ task is to ensure that the proposed research both complies with all
laws and meets the university’s standards for ethical experimentation. A proposal must gain approval from all
committees for which the research parameters apply.

Board members must sit on a minimum of one committee and a maximum of three; each committee must have a
minimum of 4 members. Committees 1 and 2 must reach a unanimous vote in order to approve a proposal.
Committees 3 and 4 may approve a proposal with no more than one “no” vote. Committees 5 and 6 must have an
odd number of members; a simple majority is sufficient to approve a proposal.
Committee 1: people age 18 and over
Committee 2: people under the age of 18
Committee 3: all other mammals (e.g., rats, rabbits)
Committee 4: all other animals (e.g., fish, worms)
Committee 5: all other biological organisms (e.g., bacteria, plant matter)
Committee 6: research that does not involve living organisms or entities

Approved studies
Three proposals were recently approved by the applicable committees.
1. A doctoral candidate was given permission to conduct a study on pain management in humans using
medication as well as alternate techniques such as massage therapy and meditation.
2. A zoology professor gained approval to conduct a series of studies on the swarming behaviors of various
animals when predators are in the vicinity.
3. A botany professor and her students received permission to catalog all living organisms native to a nearby
nature preserve that usually does not permit access by people.
Two proposals were rejected.
4. An undergraduate student was refused permission to study the physiological effects of illegal drug use in
human subjects.
5. A sociology professor’s proposal to examine the impact of sub-standard psychological treatment on
adolescents was rejected immediately when it became apparent that the professor planned to conduct
free counseling sessions during which some participants would be given care that fell below the standards
of the American Psychological Association.

Question 67

For each of the following statements, select Yes if the statement can be shown to be true using the information
provided. Otherwise, select No.
Yes No

The doctoral candidate’s proposal was reviewed by committees 1 and 2.

At least nine people voted yes on the botany professor’s proposal.

The undergraduate student’s proposal was rejected because it failed to meet the university’s ethical
standards.

Question 68

What is the minimum number of board members necessary in order to staff all six committees according to the given
rules?
6

15

24
Question 69
For each of the following statements, select Yes if the statement can be shown to be true using the information
provided. Otherwise, select No.
Yes No

At least two people rejected the undergraduate student’s proposal.

If the sociology professor amends her plan to ensure that all counseling meets the standards of the
American Psychological Association, then her proposal will be accepted.
A research study involving interactions between humans and their animal pets would require as least 14
“yes” votes in order to gain approval.

Question 70

A right cylinder is to be constructed under certain limited parameters. The height of the cylinder cannot be more than
twice the radius, nor can the reverse be true. The combined lengths of the height and the radius cannot exceed 16,
and both values must be integers.
Identify the radius and the height of the cylinder that will maximize the volume of the cylinder, given the restrictions
listed above. Make only two selections, one in each column.
Radius Height Value

10

11
Question 71

City First Quarter (Q1) Second Quarter (Q2) Third Quarter (Q3) Fourth Quarter (Q4)
Name Stores Revenues Profits Revenues Profits Revenues Profits Revenues Profits
Auburn 12 18,088 10,944 31,292 8,649 31,292 5,113 18,149 9,038
Fairview 9 14,750 4,898 5,015 1,130 1,856 728 1,707 719
Greenwood 17 20,797 8,002 9,982 1,627 3,594 2,071 2,731 1,071
Lexington 10 12,614 2,405 6,685 1,681 11,632 6,383 19,775 3,936
Milford 13 12,407 2,625 18,362 8,400 19,647 10,574 13,949 3,529
Milton 5 19,266 3,880 23,119 11,475 23,581 9,227 22,874 10,579
Mt
20 23,047 11,672 7,145 1,433 3,715 1,859 2,192 344
Pleasant
Oak Grove 5 6,753 2,848 4,457 2,095 1,738 1,017 1,512 731
Portsmouth 23 20,681 6,004 36,192 12,972 61,165 16,254 83,796 35,453
Springfield 6 17,392 5,809 13,566 3,852 17,500 8,834 6,475 2,971
Wilmington 17 25,560 15,361 31,184 17,303 32,119 14,781 11,884 4,494
York 4 7,228 3,908 5,421 1,945 4,012 1,631 6,098 2,170

FillerUp Fuels, is a regional fueling and convenience store chain that operates 389 stations in 31 cities. The table
below represents a portion of the 2011 quarterly financial records from FillerUp Fuels for selected cities, presented in
thousands of US dollars.

For each of the following statements, select Yes if the statement can be shown to be true using the information
provided. Otherwise, select No.
Yes No
Fairview had both the lowest profits and the lowest profit margin (profit/revenue) of any displayed city
during the second quarter.
Of the four cities with the fewest stores, only one saw profits steadily decrease across each of the 4
quarters.
The city with the greatest dollar increase in revenues between the first and fourth quarters increased
profits over the same period by more than $25,000,000.
Question 72

The graph shows the distribution of the US population by age (vertical axis) and gender in 2000 (dotted line) and
2009 (solid line).

Use the drop-down menus to complete each statement according to the information presented in the diagram.

Select...
In 2000, the US population of 35-to-39-year-old men was approximately of the population
of 60-to-64-year-old men.

Select...
The US population of 55-to-59-year-old women increased by approximately from 2000 to
2009.
Question 1

Statement 1: No. The only trick to this question is to recognize that it’s asking about outbreaks, not
individual illnesses. That means we can find the answer without looking beyond the first column. There
were 5 outbreaks in the first half of 2005 (any month between January and June would count as the first
half of the year) and 8 outbreaks in the second half. There were only 3 outbreaks in the first half of
2006, and and 8 outbreaks in the second half. That makes for 8 outbreaks in the first half of both years
(5 + 3 = 8) and 16 outbreaks in the second half of both years. This is exactly twice as many.

Statement 2: No. Sort by the illness column to quickly find the five highest totals. One of them caused 5
deaths, three of them caused 0 deaths, and one of them is unknown. Because of the unknown, we
cannot know if there were more deaths or not.

Statement 3: Yes. There were three total outbreaks in June of 2005 and 2006 (1 in 2005, and 2 in
2006). The number of illnesses caused by these three outbreaks were 41, 115, and 13, making for 169
total illnesses. The hospitalizations were 13, 7, and 8 in number, making for a total of 28. To find the
percent, we simply divide: 28/169 = approximately 16.5%, which is greater than 15%.

Question 2

Statement 1: $15,500. This problem asks us about information pertaining only to a specific subset of the
branches, those that saw an increase in costs during the period from 7/2/2011 to 12/31/2011. This would
be any branch with a positive Y coordinate, so we will only be looking at the nine branches above the X-
axis. From this group of 9, we need to find the one with the median change in cost during the period
beginning on 1/1/2011 (the X-axis). With 9 branches, the median will be the 5th (the physical middle). If
we count over 5 from the left or 5 from the right, we see that the median branch appears to be the point
at approximately (-3.5, 19).
Now, we are asked to determine the net Variable Cost for this branch over the entire year. This is simply
the sum of the costs for each period (in thousands), so -$3,500 + $19,000 = $15,500.

Statement 2: 47%. To find branches with a net decrease in costs it will help to understand what is
represented by each quadrant.

Quadrant I: these branches had an increase over both periods (3 branches)

Quadrant II: these branches had a decrease during the first period but an increase during the second (6
branches)

Quadrant III: these branches had a decrease during both periods (1 branch)

Quadrant IV: these branches had an increase during the first period but a decrease during the second (5
branches)
None of the branches that have an increase during both periods will have a net overall decrease, so we
can ignore the branches in Quadrant I (3 branches). All branches that have a decrease during both
periods will have a net overall decrease, so we count all branched in Quadrant III (1 branch).

Now we must look at the more complicated situations – an increase in one period but a decrease in the
other. To get a net overall decrease, we need a larger dollar decrease than increase. For Quadrant II
(decrease in first period but increase in second), we need the X-coordinate (period 1) to be larger than
the Y-coordinate (period). This appears to be the case for 3 points:
(-7, 5.5), (-15.5, 5.5), and (-16, 0.5).

Finally, for Quadrant IV (decrease in the second period but increase in the first), we need the Y-
coordinate (period 2) to have larger magnitude than the X-coordinate (period 1). This appears to be the
case for 3 points (2.5, -4.5), (0.5, -7.5), and (11, -17). For all these points, the decrease outweighs the
increase.
So in total, we have 1+3+3=7 branches out of the 15, a number slightly less than ½ or 50%. Plug it into
the calculator to get 7/15 = 0.4667 = 47%.

As an alternative, you could find all the points satisfying the expression x+y<0, or y<-x. Mentally sketch
the downward-sloping line y=-x, which runs through the origin. What points fall below the line? You’ll
find 7 points there. From here, the final calculation is the same.

Question 3

Statement 1: 50. In order to compare the differences between pairs of countries, we begin by finding the
approximate difference in 2008 cellular telephone subscribers between China and the US, and between
the US and Italy. Using our eye as a guide for 2008 (the data point farthest to the right), we see that:
China – US = 650 – 275 = 375

US – Italy = 275 – 90 = 185

The question is asking us to find approximately what percent of the difference between China and the
United States is made up by the difference between the United States and Italy. This translates to, “185
is approximately what percent of 375.” We solve this by simply doing (185 / 375) × 100 = 49.3%, which
is approximately 50%.

Statement 2: 700. The formula for percent increase is given by [(final – initial) / initial] × 100. Here, the
final value is the number United States cellular telephone subscriptions in 2008 and the initial value is the
number of cellular telephone subscriptions in 1995.

Using our eye to estimate, we have [(275 – 35) / 35] × 100 = (240 / 35) × 100 ≈ 686, which is
approximately equal to 700.
Question 4

Statement 1: No. To find the smallest ratio of "other" sources to total production, you should first sort by
the Other column. (Of course, this doesn't guarantee that the lowest ratio will be with the lowest Other
amount, but it gives you someplace to start; after all, if the total production were held constant, then
that sort would be perfect.) Look at the first row, which now contains 2001. Total production is 71.89
that year, but Other is only 5.32. Punch this into the calculator to get 5.32/71.89 = approximately 0.074
= 7.4%, which is less than 8%.

Statement 2: No. Sort by Year. Now look down the Nuclear column. You can see the numbers grow,
even if irregularly, from 6's through 7's to 8's. This represents positive correlation. (If you were able to
calculate the actual correlation coefficient, you'd find out that it is 0.946, which is very close to the
maximum of 1.)

Statement 3: No. Keep the sort on Year, for easier lookup. Now look up the Nuclear numbers for 2000
and for 2007. You get 7.86 and 8.46, respectively. You might be able already to see that the growth is
less than 10% (which would be 0.786 in absolute numbers), or you can use the calculator quickly to find
that the percent growth is approximately 7.6%, which is not more than 10%.

Question 5

In order to answer this logic problem, it might be best to start with a clear list of the constraints:

• 12 hours = total hours available


• 4 hours = total hours for walking
• Minimum 4 art or architecture activities during the 2 days
• Maximum 1 art museum per day
• Minimum of 1 beach activity during the trip
• Minimum of 1 shopping activity each day
Now we can start with the activities the family has already planned to see what has been accomplished
from the list and what has not.

On Day 1, the family has already planned 11 hours of activities, and only 1 hour of walking. Therefore
there is only 1 hour left to plan, and they could choose a walking activity if they would like. On Day 2, the
family has planned 9 hours of activities with 4 hours of walking. This means that there are 3 hours they
can use for activities but they cannot include walking.
Using these 2 constraints, we see that the only activities possible for each day are the following:

Day 1: Mirador De Colon, Montserrat, or La Pedrera

Day 2: Montserrat

Therefore, the family must choose the sightseeing trip to Montserrat on Day 2, leaving only Mirador De
Colon and La Pedrera for Day 1.

If we now look at the family preferences, we see that Mom already has her shopping on each day (Las
Ramblas and Barri Gotico) and Little Brother has his beach activity on Day 2 (Nova Icària), but Big Sister
only has 3 of her 4 art or architecture activities. Both Mirador De Colon and La Pedrera fit this category,
but Dad will not go to more than 1 art exhibit on a single day, and the family will already visit Park Güell
on Day 1. This means that they cannot visit La Pedrera as well. Therefore, the family will visit Mirador De
Colon on Day 1.

Column 1: The correct answer is A.

Column 2: The correct answer is C.

Question 6

Statement 1: Otherwise. In the last sentence of the Residency Association statement, the president
says “Such working conditions are dangerous for both patients and residents; some serious mistakes
have already been made and were caught only at the last minute by senior staff.” The president, then,
believes that the situation is dangerous and that there is at least the possibility of serious consequences
from serious mistakes made by residents.
In the last sentence of the Hospital Board statement, the spokesperson says “there is always an
experienced physician on hand to verify the diagnosis, take over the procedure, or otherwise correct any
potential errors” made by residents. The spokesperson, then, believes that there is an oversight system
in place that will “correct any potential errors” – in other words, there is not the danger of lasting harm
due to a resident’s mistake. (Note that the spokesperson doesn’t claim that the licensed physician won’t
make mistakes, but the question asks only about lasting harm caused by residents.)
Statement 2: Both Agree. The Residency Association president explicitly states that residents have
averaged 17.5 hours during the past month, so the president does believe that residents are working
more than 8 hours during a 24-hour shift. The Hospital Board spokesperson says that “residents are not
expected to work 24 hours in a 24-hour shift, nor anywhere close to that” but also that “if residents
choose to participate in many routine activities that can ably be handled by the nursing or medical staff,
that is the residents' choice.” In other words, the spokesperson is acknowledging that it is possible for a
resident to work long hours… if he or she chooses to do so. The two parties, then, both agree that
residents may work longer than 8 hours during a 24-hour shift.
Statement 3: Otherwise. The Hospital Board spokesperson states clearly that “until someone has
finished the residency, he or she is not, and cannot reasonably be considered, a licensed physician.” The
Residency Association president provides only indirect evidence that “Residents are responsible for all of
the licensed physician tasks, including intake, the ordering of tests and labs, diagnosis and treatment,
including surgery” (emphasis added). The spokesperson clearly states that no one can be considered a
licensed physician until the residency is complete. We might speculate either that the president disagrees
or that there is not enough evidence to decide whether the president disagrees. Either way, the correct
response is “otherwise.”

Question 7

Statement 1: Cannot Infer. The Residency Association indicates that the residents are assumed to work
1/3 of the 24-hour shift, or 8 hours, but the president does not claim that residents should not work more
than that amount. They claim only that they work too much. We can infer that they think they should
work less than 17.5 hours, but we do not know what they think of an 8-hour cutoff. Likewise, the
Hospital Board never indicates a specific time beyond which residents should not be able to work.
Statement 2: Both Accept. As part of their argument, the residents complain that their working
conditions are dangerous and that, as a result, serious mistakes have been made. We can infer, then,
that the residents believe the hospital is responsible for setting reasonable working hours in order to
minimize or prevent errors made by residents (among other goals). The Hospital Board indicates that
“when a resident” is working, “there is always an experienced physician on hand” in order to “correct any
potential errors.” The board, then, also believes that the hospital is responsible for minimizing or
preventing errors made by medical residents.
Statement 3: Cannot Infer. The residents state that they “are taken advantage of terribly” and cite the
fact that they are “undermined or treated poorly by senior doctors – sometimes when patients are
present” as one piece of evidence. The residents, then, agree with this statement. We might surmise that
any ethical hospital board also would not want doctors to treat medical residents poorly when patients
are present (or at any time!), but the Hospital Board’s statement never addresses this aspect of the
resident’s complaint. We do not know how the board feels about this issue.

Question 8

According to the first tab, the residents are assumed to work “one-third of the hours” in an assigned
shift. If the shift is 24 hours long, then, a resident is assumed to work for 8 hours. In that case, the
hourly wage would be $105 / 8 hours = $13.125. Rounded to the hundredths place, this is $13.13.
The resident indicates that the average time worked over the past month was actually 17.5 hours. The
hourly wage in this case would be $105 / 17.5 hours = $6.00.
The difference between the two wages is $13.13 - $6.00 = $7.13.
The correct answer is E.

Question 9
To solve this fractions problem, we must find the ratio of the number of total boxed packed by the first-
shift to the total number of boxes packed by both shifts together. From the information given, we know
that there were 2/3 as many first-shift workers as second-shift workers and, inverting the second
fraction, we know that each first-shift worker packed 3/4 as many boxes as each individual second-shift
worker. From here, multiplying the ratio of workers by the ratio of work per individual gives the fraction
of total first-shift boxes relative to the second shift. This is done as (2/3) × (3/4) = 6/12 = 1/2. Thus, the
first shift packs half as many boxes as the second shift. We can compute the first-shift boxes relative to
the total by:
(first-shift fraction) / (first-shift fraction + second-shift fraction) = 1 / (1+2) = 1/3.
Thus the first shift does 1/3 of the total work. We must look for two numbers in the table that are related
by a factor of 3. The only two numbers are 12 and 36, meaning that the first shift packed 12 boxes, and
the total number of boxes packed by both shifts was 3 × 12 = 36.
Alternatively, one could use numbers to establish the relationship between the number of total boxes
packed by the first shift and the number of total boxes packed by the two shifts together. We use our
fractional ratios to choose smart numbers and assign 2 workers to the first shift, 3 workers to the second
shift, 3 boxes per individual on the first shift, and 4 boxes per individual on the second shift. This gives:
Total First-shift Boxes = (2 workers) × (3 boxes per worker) = 6 boxes
Total Second-shift Boxes = (3 workers) × (4 boxes per worker) = 12 boxes
Total Boxes Overall = 12 boxes + 6 boxes = 18 boxes.
Again, from this we can derive that the ratio of first-shift boxes to the total boxes packed is 6/18 = 1/3.
Column 1: The correct answer is C.
Column 2: The correct answer is E.

Question 10
Statement 1: 60% and 65%. A person who was 30 in 1999 was 39 in 2008. Therefore, to answer this
question, we need to multiply the probability that a person who was 30 in 1999 had health insurance by
the probability that a person who was 39 in 2008 had health insurance. (The assumption of
independence allows us to multiply these probabilities; in reality, these conditions are almost
certainly not independent.)
79% of people aged 25-34 had health insurance in 1999, so a person who was 30 in 1999 had a 79%
chance of having health insurance. Similarly, a person who was 39 in 2008 had an 80% chance of having
health insurance.
The probability we want is 79% × 80%.
0.79 × 0.80 = 0.632 = 63.2%, between 60% and 65%.
Statement 2: 1,200,000 less. The easiest way to answer this question is to calculate the number of 35-
44 year olds who had health insurance in 1999 and 2008. The total US population is given in the
information under the graph. The population in 1999 was 276,804,000; the population in 2008 was
301,483,000.
If 35-44 year olds were 20% of the population in 1999, and 85% of 35-44 year olds had health
insurance, then the total number of 35-44 year olds who had health insurance in 1999 was:
276,804,000 × 0.85 × 0.20 = 47,056,680 ≈ 47 million
Similarly, 80% of 35-44 year olds had health insurance in 2008, and 35-44 year olds represented 20% of
the population:
301,483,000 × 0.80 × 0.20 = 48.237,280 ≈ 48.2 million
The question said approximately, so we don’t have to find the exact difference. 48.2 – 47 = 1.2. We’re
talking about millions of people, so there were approximately 1,200,000 fewer 35-44 year olds insured in
1999 than in 2008.

Question 11

The conclusion of the argument is that the Dante 5000 is more reliable than the company’s other stone-
cutting machines and the premise is that there have been fewer customer complaints about the Dante
500 than about the company’s other industrial stone-cutting machines over the last six months.
This argument assumes a number of things! Do breakdowns in unreliable machines typically occur within
the first six months? Have enough Dante 5000’s been sold to make a reasonable comparison with the
other stone-cutting machines?
Option A: The Dante 5000 is the most expensive stone-cutting machine produced by the company.
The argument is not about cost. This is out of scope.
Option B: There are other stone-cutting machines that are considered more reliable than the Dante 5000.
The argument is only about this company’s stone-cutting machines. Other machines may be from other
companies, so this is out of scope.
Option C: The Dante 5000 performed very well in initial testing.
The argument is reliability measured in terms of breakdowns, not about performance. This is out of
scope.
Option D: Stone-cutting machines usually break down very quickly under industrial use if they are not
reliable. - Strengthens
This option directly strengthens the conclusion that the Dante 5000 is reliable than the company’s other
machines because it makes it more likely that six months would be enough time to see breakdowns if the
machine were unreliable. Note that this statement does not plug all of the holes in the argument! It just
makes this weak argument slightly stronger.
Option E: Very few customers have purchased a Dante 5000. - Weakens
This option directly weakens the conclusion by providing a good alternative interpretation of the low
number of customer complaints. If very few customers have bought the machine, then a small number of
complaints could actually represent a high rate (percent) of complaints. This choice points up the classic
difference between absolute numbers and percents.
Option F: The Dante 5000 employs a new technology that is more precise than that used by our previous
stone-cutting machines.
The argument is concerned with reliability measured in terms of breakdowns, not about precision. This
choice is out of scope.
Column 1: The correct answer is D.
Column 2: The correct answer is E.

Question 12

Remember that R × T = D. We are given that the distance = 675, but only that the time for train A
is t and that the time for train B is t – 3. So we know that RA × t = 675 and RB × (t – 3) = 675.
So RA = 675/t and RB = 675/(t – 3), but we don’t have enough information to solve for either rate,
because there are only two equations and three unknowns.
What we do know though, is that the rate, or speed, of train A must be less than the speed of train B and
that both rates must be in the chart.
So we can test the smaller numbers as potential speeds of A. Solve for potential t’s and (t – 3)’s and then
test the (t – 3)’s until we find one that yields a speed that is in the chart. Use the calculator, of course, to
move quickly through these computations.
Potential value for Implied Implied value of B’s
Implied value of t
A’s speed in mph value of t– 3 speed in mph
t= 675/45.75 = = 675/11.754 = 57.427
45.75 t – 3 = 11.754
14.754 not on list
= 675/10.5 = 64.286
50 t = 675/50 = 13.5 t – 3 = 10.5
not on list
= 675/9 = 75
56.25 t = 675/56.25 = 12 t–3=9
yes on list! Stop!
63
67.5
75
Since there can be only one set of answers, we can stop here.
Column 1: The correct answer is C.
Column 2: The correct answer is F.

Question 13

The passage as a whole strongly suggests that few cartels last more than 4 or 5 years because of the
economic incentive that members have to cheat.
Option A - No cartel will last more than 100 years.
This is much too strong of a statement to be logically inferred from “very few known cartels have lasted
for more than 4 or 5 years.” It could be that one or more known (or unknown) cartels has indeed lasted
100 years, even though only 4 to 5 years is typical.
Option B - As long as the members do not cheat, a cartel cannot be broken.
This is too strong of a statement to be inferred from “The main issue is that the members of a cartel all
have an incentive to cheat and cut prices just a little in order to maximize their individual profits at the
expense of profits of the cartel as a whole.” There could be other issues. Perhaps new suppliers that are
not part of the market enter the cartel, or perhaps the cartel’s product is made technologically irrelevant.
Option C - An effective system for preventing members from cheating would not increase the
likelihood that a cartel would survive in the long-term. – Correct FALSE
The passage says that the main issue is that the members of the cartel have an incentive to cheat, so an
effective system for preventing cheating WOULD likely increase the likelihood that a cartel would survive
in the long run.
Option D - Private cartels are not legal in most countries.
The passage does not discuss legality. Although this statement is likely true, it is not something that can
be inferred from the passage.
Option E – Cartels are inherently unstable and likely to fail in the long run. – Correct TRUE
This can be directly inferred from “very few known cartels have lasted for more than 4 or 5 years” and
“The main issue is that the members of a cartel all have an incentive to cheat and cut prices just a little
in order to maximize their individual profits at the expense of profits of the cartel as a whole.” The
economic incentive to cheat is inherent in the nature of a cartel and makes it inherently unstable.
Option F – It is extremely difficult for competing firms to agree to fix prices, marketing, and
production in the formation of a cartel.
This choice is tempting, but the passage does not discuss the difficulty of forming a cartel agreement.
Although this statement is likely true in the real world, it is too extreme to infer language such as
"extremely difficult" from the passage.
Column 1: The correct answer is E.
Column 2: The correct answer is C. It is extremely difficult for competing firms to agree to fix prices,
marketing, and production in the formation of a cartel.

Question 14

The key to answering this question is finding the relationship between X and Y. First, create an equation
that matches the information provided.

Revenue from widget sales will equal the number of widgets sold times the price per widget. Let w be the
original number of widgets sold and let p be the original price per widget. “X%” is the same as X/100;
likewise, “Y%” is Y/100.
If the revenues are equal for the two months, then:

wp = (Y/100) × (w) × (X/100) × (p)

Notice that both sides of the equation contain w and p. Cancel out w and p (which are non-zero, by the
logic of the real world, so you’re allowed to divide them away).

wp = (Y/100) × (w) × (X/100) × (p)


1 = (Y/100) × (X/100)

You can go further (to prove that XY = 10,000), but let’s stay in “percent” land. What we have so far is
that (Y/100) and (X/100) multiply together to 1. That is, they are reciprocals of each other.
So let’s save time. Divide every answer choice by 100, to convert it to the decimal equivalent of a
percent. We get
0.50
0.625
0.75
1.5
1.6
1.8
Now, to determine which pairs multiply together to 1, notice that you’ll need to pick one number smaller
than 1 and the other number larger than 1. Which variable gets which? Since the factory “raised” the
price to X%, we know that X% must be bigger than 100%, so X must be the one bigger than 1.
Next, to check reciprocals quickly, see whether there’s a quick fraction equivalent of each of these
decimals. Fortunately, there is!
0.50 = ½
0.625 = 5/8
0.75 = ¾
1.50 = 3/2
1.60 = 8/5
1.80 = 9/5
Now it’s easy to spot the reciprocals. Know your eighths! (1/8 = 0.125, etc.)
5/8 × 8/5 = 1, so the correct percents are 62.5% and 160%. Again, X must be 160 and Y must be 62.5.
Column 1: The correct answer is E.
Column 2: The correct answer is B.

Question 15

Statement 1: Brazil. In order to solve this problem, you need to know that the definition of the median
is the middle number in a set.
Since there are 15 countries in the chart, the one with the 8th largest national debt has the median
national debt. To see this quickly, hold your finger or a piece of paper on the right edge of the chart,
start moving it to the left, and count off countries as the piece of paper touches their bars until you get
to the eighth longest bar, which is Brazil’s.
Statement 2: between 50% and 70%. Use the lines on the graph to help with counting countries that
fall into the ranges or “bins” given in the answer choices. Notice that the percent ranges are the
midpoints between the lines shown.
Since there are only two countries, Italy and Japan, with national debt as a percentage of GDP greater
than 70%, we can quickly eliminate answers D and E. Five countries (United States, India, Germany,
France, and Canada) have percentages between 50% and 70%. The United Kingdom, Spain, Brazil, and
possibly South Korea have values between 30% and 50%, for a total of 3 or possibly 4 countries in that
range, but even 4 is less than the 5 countries between 50% and 70%. Australia, China, Mexico, and
probably South Korea have percentages between 10% and 30%, for a total of 4 countries. Russia is the
only country below 10%. So the answer must be between 50% and 70%.

Question 16

Statement 1: No. First, compute the "turns" (as defined in the statement) of AV materials for 2004 (the
date in the statement) by dividing "AV materials lent out to public" by "AV materials in collection" in the
2004 row. 6,683 / 2,456 = 2.72.
Now the question is this: is there any year that has a smaller number? Sort by the numerator of the ratio
you're examining, "Lent out to public AV materials," and look at the top row. The year in the top row is
2005, and the "Lent out to public AV materials" number is 6,059 (the smallest of any year).
Does that generate a smaller ratio? Try it: 6,059 / 2,383 = 2.54, which is smaller than the 2004 ratio.
You could tell in advance, if you looked closely, because 6,059 represents a bigger % decline from 6,683
(almost 10%) than 2,383 does from 2,456 (about 3%).
So the AV turns in 2004 are not the lowest among years listed.
Statement 2: No. First, re-sort by "Year" if necessary, so that your table is in chronological order.
Measure the percent decline in "In collection Volumes": (37,464 - 42,760) / 42,760 ≈ -12%, or a 12%
decline. Did any other characteristic listed in the table experience a greater percent decline? Scanning,
we should see the first column ("Libraries Number") pop out at us: it fell from 508 to 351, a decline we
should be able to estimate as 30% without using the calculator by rounding to 500 and 350 (the
calculator puts the decline as approximately 31%).
So the listed characteristic that experienced the greatest percent decline was not volumes in collection.
Statement 3: No. For each year in question (2002 and 2003), compute the ratio of "In collection Books"
to "Users Registered."

2002: 36,671 / 4,290 ≈ 8.55


2003: 35,371 / 4,206 ≈ 8.41
Notice that both raw numbers fell from 2002 to 2003 (books by about 3.5%, and users by about 2.0%),
and the ratio only fell very slightly. So we should go ahead and use the calculator here, rather than risk
error.
The number of books in the collections per registered user did not rise from 2002 to 2003.

Question 17

You are asked to figure out and apply these linguistic constraints to possible words. Let’s start with the
classification of vowels as “brutish” (a, o, and u) and as “fragile” (e and i).
Kurtish has stricter rules about the separation of these vowels—only one kind can appear within any
word. Since this is a simple constraint, let’s apply it first. Which words fail this constraint?
calzral – only a, brutish – passes
fjp – NO vowels, but that’s okay (see note below) – passes
aphueitse – both brutish (a and u) and fragile (e and i) – fails, can’t be Kurtish
brushmen – both brutish (u) and fragile (e) – fails, can’t be Kurtish
(don’t be fooled by the fact that brushmen sounds okay in English, though it’s not an English word)
qudxatroua – only brutish (a, o, and u) – passes
hzziigri – only fragile (i) – passes
So we know that our two answers must be in the middle, aphueitse and brushmen. One of these could
be Laeglish; the other cannot. Now we have to apply the second constraint, the one about Laeglish:
every consonant or group of consonants can only “directly touch” vowels of one type or the other. Let’s
compare:
aPHueiTSe
– the PH touches a and u, which are both brutish
– the TS touches i and e, which are both fragile
This passes the Laeglish test. Double-check the other word:
BRuSHMeN
– the BR only touches u, but the SHM touches u (brutish) and e (fragile). That breaks the Laeglish rule.
So aphueitse could be Laeglish but not Kurtish; brushmen could not be either.
Column 1: The correct answer is C.
Column 2: The correct answer is D.

Note that fjp does not have any vowels as defined above, but no constraint demands that a word in
either language contain vowels! In fact, a nod is given to the possibility of zero vowels within a word with
the language “according to the vowels it [a word] contains, if any” (emphasis added). Don’t apply outside
knowledge inappropriately here (you expect words to have vowels).

Question 18

Statement 1: Canada. This problem asks us to find the country with the highest standard deviation
among all years. Standard deviation is a measure of the overall distance of the set from the mean value,
or the “spread” of the set. Judging by eye from the plot, we see that Switzerland and the Netherlands are
much closer to their mean values across all years than Sweden and the Canada, so we eliminate
Switzerland and the Netherlands. Notice that Sweden and Canada start out at the same value of zero and
then both trend upwards in the same way over the years. However, Canada is always higher than
Sweden, meaning the overall spread of the set of values for Canada is larger.
Since they start at the same value and Canada always trends higher, the Canadian set is much more
spread out, and therefore Canada has the larger standard deviation.
Statement 2: 1,500. To solve this problem, we find the ratio of the two countries and then multiply by
100 to achieve a percent. This is given by:
Percent = [(Sweden in 2000) / (Netherlands in 2000)] × 100
Finding the year 2000 along our x-axis, we use our eye to judge the approximate values of production for
the Netherlands and Sweden in 2000. The Netherlands produced approximately 1 million toe of nuclear
energy in 2000 and Sweden produced approximately 15 million toe of total nuclear energy in the same
year. Plugging these values into the formula above:
Percent = (15 million / 1 million) × 100 = 15 × 100 = 1,500.

Question 19

According to the table on the Funding Sources tab, three teams have a Percent Funded figure exceeding
100%: Gymnastics, Soccer, and Track and Field. For each of these, we can calculate the amount of extra
money generated by the respective team. The first tab indicates that there are four funding sources and
that individual endorsements are not part of the overall team calculations, leaving three funding sources:
TV rights, team sponsors, and government funding. These three sources are all detailed in the second
tab. (Note that none of the three will receive government funding because all three already exceed 100%
of their own expenses.)
The gymnastics team has a total of 1,100 + 890 = 1,990 in funding and this represents 115% of the
team’s monetary needs, or 1,990 = 1.15x. The actual amount needed, then, is 1,990 / 1.15 = 1,730, and
the surplus is 1,990 – 1,730 = 260. (Note that we can round our calculations because the problem asks
for the approximate level of surplus funding.)
The Soccer team has a total of 1,800 + 1,300 = 3,100 in funding and this represents 126% of the team’s
monetary needs. The actual amount needed, then, is 3,100 / 1.26 = 2,460, and the surplus is 3,100 –
2,460 = 640.
The Track and Field team has 1,000 + 345 = 1,345 in funding and this represents 103% of the team’s
monetary needs. The actual amount needed, then, is 1,345 / 1.03 = 1,306, and the surplus is 1,345 –
1,306 = 39.
The total surplus is 260 + 640 + 39 = 939, or approximately 940.
The correct answer is A.

Question 20

Statement 1: No. The second paragraph of the Article tab indicates that “an individual team is
considered underfunded if its funding (not including government sources) covers less than 95% of the
team’s expected expenses.” There are 9 teams shown in the Funding Sources tab. Of these, only 4 are
funded at less than 95% of expected expenses (basketball, boxing, equestrian, and volleyball).
Statement 2: Yes. The Article tab tells us that surplus funding will be returned to the committee and
reallocated among the teams who have not reached 100% funding. The government is willing to provide
up to 5% of expenses to those teams who fall short, but it will not have to provide the full amount in all
cases because there is a surplus among three teams that will be able to be distributed among the “short”
teams. Therefore, the government will be paying some reduced amount. (In fact, the surplus of the three
teams is larger than the shortfall of the other 6 teams, so the government will not be responsible for any
funding at all.)
Statement 3: No. It is true that the absolute amount of government funding is higher for the equestrian
team (43) than for the basketball team (40). But the question asks about the funding as a percentage of
total funds needed for that team. The passage tells us that the government will pay up to a maximum of
5% for any team that cannot meet its funding needs – but the government will only pay the full 5% if the
team’s own funding is below 95% of its needed funding. Both the basketball and equestrian teams are
below 95% of full funding, so both teams qualify for the maximum 5% in government funding.

Question 21

To start, we need to calculate how much funding each team needs. If TV rights will increase by 15% for
each sport, we need to recalculate the amount of funding from TV rights, and determine the new amount
by which the team falls short of the goal (if any). Then we need to determine whether that is more or
less than 5% of needed funding.
Statement 1: Yes. Prior to the change, the basketball team had secured 400 + 250 = 650 in funding
from TV rights and team sponsors, and this represented 82% of the team’s needed funding. Therefore,
the team needed a total of 650 / (82/100) = 793 in funding.
Now, the team will earn 400 × 1.15 = 460 in TV rights, or an increase of 60, so the team will have a total
of 650 + 60 = 710 in funding. Calculate the percentage that this represents of total funding: 710 / 793 =
90%. The team is still more than 5% short, so it still qualifies for the maximum 5% government funding.
Statement 2: No. Prior to the change, the boxing team had secured 130 + 185 = 315 in funding from TV
rights and team sponsors, and this represented 92% of the team’s needed funding. Therefore, the team
needed a total of 315 / (92/100) = 342 in funding.
Now, the team will earn 130 × 1.15 = 149.5 in TV rights, or an increase of 19.5, so the team will have a
total of 315 + 19.5 = 334.5 in funding. Calculate the percentage that this represents of total funding:
334.5 / 342 = 98%. The team is less than 5% short, so it no longer qualifies for the maximum 5%
government funding.
Statement 3: Yes. Prior to the change, the volleyball team had secured 100 + 130 = 230 in funding from
TV rights and team sponsors, and this represented 86% of the team’s needed funding. Therefore, the
team needed a total of 230 / (86/100) = 267 in funding.
Now, the team will earn 100 × 1.15 = 115 in TV rights, or an increase of 15, so the team will have a total
of 230 + 15 = 245 in funding. Calculate the percentage that this represents of total funding: 245 / 267 =
92%. The team is still more than 5% short, so it still qualifies for the maximum 5% government funding.

Question 22

Statement 1: 1775. The graph gives you enough information to compute the number of characters in
each document. Examine the labels of the columns:
1. Characters per word = Characters ÷ Word
2. Words per sentence = Words ÷ Sentence
3. Sentences per paragraph = Sentences ÷ Paragraph
4. Paragraphs
If you multiply (1) and (2) together, the Words cancel, and you're left with "Characters per sentence."
Keep multiplying all the way across, and you cancel out all the denominators! You're left with Characters.
Now, one good and safe way forward is simply to estimate the numbers for each document (A through
D), and multiply across, using the calculator. That might take a little time, but it's secure and not too
time-consuming. Here's a set of sample estimations and results:
(A) 5.22 × 45 × 1.5 × 26 ≈ 9,161
(B) 5.17 × 40 × 4.4 × 15 ≈ 13,648
(C) 5.06 × 38 × 1.1 × 32 ≈ 6,768
(D) 5.04 × 150 × 1 × 2 ≈ 1,500
So document B has the most characters. Checking against the text note, you can find that document B
was published in 1775.
Even if you estimate the numbers slightly differently, you'll still come out with document B as the clear
winner.
You can avoid a little computation by not computing out D's numbers, but if you notice, A and B are
proportionally relatively close. A slight shift in a few points for each document, relatively unnoticeable,
could cause the total characters of A to exceed those of B. The safe move is to estimate each number
and multiply out.
Statement 2: the ratio of words to sentences.
"The ratio of characters to words" is another name for the "Characters per word" metric in the first
column. Likewise, the other ratios in the answer choices are simply different names for the respective
columns. So the question is this: Column 1 is most likely to be negatively correlated with which other
column?
Helpfully, column 1 is in ABCD order. What order are the other columns in?
2. DABC
3. BACD
4. CABD
So they all preserve something of the original order, but notice that column 2 has the most drastic move
(D moves to the very front). Moreover, in column 2 the letters besides D are all bunched up together,
making their relative order less important; meanwhile, D has leapt out to a much greater number (150)
than any of the others (which are somewhere between 35 and 45). This makes the effect of D's move
much greater. (Yes, column 3 experiences something akin to this effect, but it's B that's out in front, and
since B was already toward the beginning, it's less of a reversal.)
So the second column, with Words per sentence, that is most likely to be negatively correlated with the
first column (Characters per word). You aren't responsible for computing the actual correlation coefficient
until you get to b-school, but such computations bear out this result (the actual correlation coefficients
between the first column and the others are -0.58, 0.43, and 0.33, respectively).

Question 23

Statement 1: No. Sort by column “1996” to find the range (maximum – minimum) for that year. The
maximum value is 137.407 and the minimum value is 120.99, so, using the calculator, the range is
137.407 – 120.99 = 16.417. For the range to be greater than 15% of any single nation in 2000, the
range must be greater than 15% of the maximum blood pressure posted in 2000. Sort by column “2000”
to find the maximum value; 136.785. Use the calculator to compute 15% of this value; (0.15) (136.785)
= 20.51775. Because the range in 1996 is less than 15% of the maximum value in 2000, (16.417 <
20.51775), this statement cannot be shown to be true.
Statement 2: No. Sort each column independently, noting the country with the highest blood pressure for
each year. The results are:
1980: Finland
1984: Finland
1988: Serbia
1992: Serbia
1996: Serbia
2000: Serbia
2004: Niger
2008: Niger
So the nation that most frequently ranked highest in blood pressure was Serbia, which means that this
statement cannot be shown to be true.
Note that if the sorting was performed chronologically, as shown above, then we could have stopped
after 2004, since at this point it was certain that Serbia had the highest blood pressure most frequently
and not Finland.
Statement 3: Yes. This prompt is asking us to find all nations that had an average male blood pressure
below 125 mmHg in both 1980 and 2008. We can begin by sorting either column and identifying the
nations that fit the criteria. For example, sorting the table by column “1980” shows that Papua New
Guinea, Cambodia, India, Bhutan, Thailand, Vietnam, Nepal, and Maldives all had blood pressures below
125 mmHg in 1980. Now, sorting on “2008” shows that of these nations, only Papua New Guinea,
Cambodia, Turkey, and Thailand remained below 125 mmHg in 2008. Therefore four nations fell below
125 mmHg in both years; this statement can be shown to be true.

Question 24

Although you could draw a Venn Diagram or a Double-Set Matrix to answer this question, there is a
formula we can use to save time. Suppose there is an overlap between two groups, group X and group Y.
The total number of items equals the number of items in group X, plus the number of items in group
Y, plus the number of items in neither, minus the number of items in both – because you double-counted
them by adding X and Y. So here’s the formula:
X + Y + Neither – Both= Total
We can say the same for this problem:
Spanish + French + Neither – Both = Total
We know the total number of students is 350 and the number of students currently enrolled in Spanish is
230. Furthermore, we actually know that none of those 350 students is taking neither French nor
Spanish.
230 + French + 0 – Neither = 350
French – Neither = 120
We need to find 2 choices that differ by 120, with the larger number representing the total number of
students currently enrolled in French, and the smaller number representing the number of student
enrolled in both French and Spanish. 260 and 140 are the only numbers in the list that have a difference
of 120.
Column 1: The correct answer is E.
Column 2: The correct answer is A.

Question 25

1st part
To calculate the interest rate at which the two options have equal total interest payments, figure out how
much interest would be paid on the $240,000 loan at 5% and on the $190,000 loan at 4%.
5% of $230,000 = 0.05 × $240,000 = $12,000 <- use the calculator to do this quickly
4% of $190,000 = 0.04 × $190,000 = $7,600
For the total interest payment on Option 1 to be equal to the total interest payment, x, on Option 2, the
interest on the $50,000 loan plus the interest on the $190,000 loan must be equal to the interest
payment on the $230,000 loan. So
$12,000 = $7,600 + x
x = $12,000 – $7,600 = $4,400
Since you know the total interest payment for the $50,000 loan, you can compute the interest rate r%.
Interest rate × principal = interest payment
Interest rate = interest payment / interest rate
r% = $4,400 / $50,000 = 0.088 = 8.8%

2nd part
To calculate the interest rate at which the interest payment on the $190,000 loan at 4% would be equal
to twice the interest payment on the $50,000 loan at r%, first compute the interest payment on the
$190,000 loan:
4% of $190,000 = 0.04 × $190,000 = $7,600
If the interest payment the $50,000 loan is equal to half the interest payment on the $190,000 loan, then
you have:
Interest payment on $50,000 loan = $7,600 / 2 = $3,800
Since you know the total interest payment for the $50,000 loan, you can compute the interest rate r%.
Interest rate × principal = interest payment
Interest rate = interest payment / interest rate
r% = $3,800 / $50,000 = 0.076 = 7.6%

Column 1: The correct answer is E.


Column 2: The correct answer is D.

Question 26

Statement 1: True. Sort by total ore tonnage. The two countries with the greatest ore tonnage are the
United States (2,464.8mm) and Australia (1,939.5mm). In order to calculate the exact values for the zinc
and lead, we can multiply the percentages for each by the millions of metric tons of ore. There is a
shortcut though: the problem asks us specifically to determine whether the zinc deposits are more than
1.5 times greater than the lead deposits. In the United States, lead deposits make up 2.38% of the ore.
Multiply this number by 1.5: 2.38 × 1.5 = 3.57. The percentage for zinc grade is 3.88, which is greater
than 3.57, so for the United States, it is true that zinc deposits are more than 1.5 times lead deposits.
For Australia, lead deposits make up 4.39 percent. Multiply 4.39 × 1.5 = 6.59, which is less than the
actual zinc grade percentage of 6.97 in Australia. In this case, too, zinc deposits are more than 1.5 times
lead deposits.
Statement 2: True. Sort by the Zinc Grade column. There are 12 countries, so the median will be the
average of countries 6 and 7 (when sorted by Zinc Grade). The United States is #6, with 3.88% zinc
grade and India is #7, with 4.06% zinc grade. Note: we do NOT need to calculate the actual value of the
median. Any countries at or below 3.88% zinc grade are below the median, and any countries at or
above 4.06% zinc grade are above the median. There are no countries between these two numbers
(logically, there will never be, because these two figures are side by side on the list with nothing in
between, by definition).
Next, sort by the Lead Grade column. There are 6 countries with a Lead Grade percentage of 2 or greater
(Germany, United States, Canada, Mexico, Russia, and Australia). Of these 6, how many have a zinc
grade value of 3.88% or less? Three: Germany, United States, and Russia. It is the case, then, that
exactly half (3 of 6) of the countries with a lead grade percentage of 2 or greater also have zinc grade
percentages below the median.
Statement 3: True. The term “range” refers to the difference between the largest number and the
smallest number in a data set. First, note that both Total Ore Tonnage and Average Age are given in
millions of years, so we can ignore the “millions” portion of each number and simply use the actual
numbers shown in the table. Sort the table by Total Ore Tonnage. The range is 2,464.8 – 336.2 =
2,128.6. Next, sort by Average Age. The range is 1,791 – 15 = 1,776 (or notice that the larger number
here, 1,791, is already smaller than the range of Total Ore Tonnage, which is more than 2,000).

Question 27

You’re given a long paragraph of text followed by a question. In this circumstance, it’s usually beneficial
to read the question stem first; then you’ll have a good idea of which information will be most important
while reading the passage. The question indicates that there is at least one activity that non-prescription
medication manufacturers in Country X are required to perform, and there is at least one that is not
permitted. You need to figure out what those two things are.
The initial sentence indicates that Country X has instituted some new regulatory requirements (in simpler
speech: laws) for non-prescription medication manufacturers. If companies import any components from
other countries, they have to disclose those components, or tell the government what those components
are. The second sentence details two additional requirements: (1) a company must use at least two
different suppliers for any component, and (2) for certain medications (not all, only some), a company
can’t rely on just one country for more than 70% of any single component. Note something else
important about this sentence: item (1) does not mention anything about where the supplier is located.
In other words, a company could have two suppliers in Country X, or one in Country X and one in
another country. The final sentence explains three reasons why these regulations were implemented: to
minimize contamination risks and / or financial dependencies on other countries, and to maintain a
steady supply of critical drugs.
Because you know that the question asks what activities are or are not allowed, most of your attention
should be focused on the first two sentences – but you do still want to read this final sentence, though
you may choose not to take detailed notes on it. One version of the notes might look like this:
X: laws for NPM coms
(1) disclose comps
(2) 2+ suppl per comp
(3) for some meds, 1 comp < 70% from 1 country
Why? 3 reasons
Next, you examine the answer choices. What must these manufacturers do and what must they NOT do?
Buying more than 70% of a component from one country: This choice is very tempting; the passage did
mention a 70% figure. Be sure to verify the details in your notes. That requirement was true
for some medications, but not all. If you’re talking about a medication that doesn’t fall into this category,
then the manufacturer is not required to follow the 70% rule. This may be done but does not always
have to be done; it is not the correct answer for either part of the question.
Importing components from other countries: The passage is concerned with what rules the
manufacturers must follow when they do import from other countries, so companies are permitted to
import, but nowhere does the passage indicate that manufacturers must import from other countries.
This is not the correct answer for either part of the question.
Reporting to authorities the foreign components used: The first sentence said that these manufacturers
“must disclose all sources of components that are imported from other countries.” In other words, the
manufacturers are required to report any of the foreign components they use. This is the answer to the
first part.
Using a component for which only one supplier exists outside of Country X: This choice is very tempting;
be careful not to make assumptions that do not have to be true. The second sentence does indicate that
companies must have at least two suppliers for any individual component, but the regulations do not
specify where those suppliers must be. One or both could be in Country X. Nor does the paragraph
indicate that this 2-supplier requirement applies only to foreign-sourced components; the sentence says
only “for any individual component.”
Manufacturing a drug whose components are all from a single supplier: The second sentence does
indicate that a manufacturer must have at least two suppliers for any individual component. Even if a
drug is made from one single component, the manufacturer must have at least two suppliers of that
component. This is the answer to the second part.3
Column 1: The correct answer is C.
Column 2: The correct answer is E.

Question 28

Statement 1: Conceptual Entity. First, find the box labeled “Body Location or Region.” It’s on the right
side. The note tells you that an “is a” relationship is indicated by a solid rectilinear connector between a
higher (parent) node and a lower (child) node. So, “Body Location or Region” has an “is a” relationship
with “Spatial Concept,” which is its parent. In other words, Body Location or Region is a specific kind of
Spatial Concept. (Note that “Body Space or Junction” is a sibling (on the same level), so there is no “is a”
relationship with “Body Location or Region.”)

Unfortunately, “Spatial Concept” is not an answer choice. However, you are told that “is a” relationships
are inherited by grandchildren, great-grandchildren, etc. In other words, you can keep traveling up the
tree. Body Location or Region is a specific kind of Idea or Concept, a specific kind of Conceptual Entity,
and a specific kind of Entity as well. Of these, the only choice among the answers is Conceptual Entity.

Body Location or Region is associated with other nodes via curved dotted connectors, but those do not
represent “is a” relationships.

Statement 2: Body Substance. The key words to notice are “spatially and functionally related.” These
are two types of curved dotted connections between nodes. So the fast way to narrow down choices is to
isolate nodes that are shown with two curved connections, then check the connection type.
Body Substance: has a produces relationship and a location of relationship with two other nodes. From
the second hierarchy, you can see that produces is a functional relationship, while location of is a spatial
relationship. This is the answer.

The only other nodes that have two curved relationships shown are these:
1) Body Part, Organ, or Organ Component
2) Body Location or Region.

Body Part, Organ, or Organ Component: has a part of relationship and a conceptual part of relationship
with two other nodes. Part of is a physical relationship, while conceptual part of is a conceptual
relationship. No good.

Body Location or Region: has an adjacent to relationship and a location of relationship. These are both
spatial relationships. No good.

Question 29

To solve this logic problem, piece the clues together to see what restrictions they put on the final
positions of runners. Work from the most restrictive clues to the least restrictive. The easiest clue to use
first is that runner D finishes in fourth place. Graphically, draw this as:
___ ___ ___ _D_ ___ ___ ___
You know that runner C finishes in the top 50% but not in the first position, which gives two possibilities:
___ _(C)_ _(C)_ _D_ ___ ___ ___
You also know that A and E finish consecutively, which means they will always finish back-to-back,
though you do not know in which order (A then E or E then A). Therefore, runners A and E could finish
1/2, 5/6, or 6/7. However, runner B finishes before F but after A, meaning B must also finish behind E,
since E is consecutive to A. From this you can deduce that runners A and E must finish in slots 1/2, or
there would not be enough spots left for B and F. As runner A does not appear as an option in the table,
you must select E as the runner that could have finished first.
Finally, the above tells you that the last 3 slots as taken by B, F, and G. Knowing that B finishes before F
leaves 3 distinct possibilities for this placement:
___ ___ ___ _D_ _B_ _F_ _G_
___ ___ ___ _D_ _B_ _G_ _F_
___ ___ ___ _D_ _G_ _B_ _F_
Therefore, the runners that could have finished fifth place are B and G. Since G is not in the chart, select
B as your answer.
Column 1: The correct answer is E (fourth entry).
Column 2: The correct answer is B (first entry).

Question 30

Statement 1: No. We cannot show definitively that the customer will not order the syringes from this
supplier. The customer does not state that she will not order the syringes if she does not receive a
discount specifically on the syringes. In fact, she states that her boss “might be willing” to place the order
if the company receives a volume discount (and she doesn’t specify that this must be for the syringes);
the supplier did offer a volume discount on the pumps. It’s possible, then, that she will also order the
syringes from this supplier.
Statement 2: No. The supplier does claim that his syringes will “typically” last longer than the guaranteed
20 injections. If a syringe lasts for 30 injections, that would be (30-20)/20 = 50% more injections than
guaranteed. If a syringe lasts for 40 injections, that would be (40-20)/20 = 100% more injections than
guaranteed. The supplier claims that his syringes typically allow for 50% to 100% more injections than
the base level guaranteed.
Statement 3: Yes. In the 2nd email, the customer indicates that she is unhappy about the pricing and
then states “If we were able to get a volume discount, though, he [the boss] might still be willing to give
you the order.” This last part implies that they are already thinking seriously about switching to the other
supplier.

Question 31

In this “Except” question, we will need to find the “odd one out” answer. Four of the answer choices
describe negotiating tactics used by the customer. The fifth one (the correct answer) does not.
(A) Incorrect. The customer attempts to indicate the value of her business by pointing out to the supplier
that her company is “ordering more than 1,000 pumps.”
(B) Incorrect. The customer specifically asks for a large-volume discount.
(C) Correct. The supplier offers a concession, but the question asks us to find negotiating tactics used by
the customer, not the supplier. The customer asks for a volume discount on the pumps; the supplier
offers a volume discount on the entire order (if the customer orders more).
(D) Incorrect. The customer both tells the supplier to “hold off on the syringe order” and mentions that a
volume discount might make her boss “willing to give you the order.” This implies that she is willing to
consider walking away from the deal if needed.
(E) Incorrect. The customer states that her boss is “upset” about previous price increases.
The correct answer is C.

Question 32

Statement 1: No. Email #1 indicates that the 20-injection syringes sell for $20 per 50 syringes, or $20 /
50 = $0.40 per syringe. Each syringe is guaranteed for 20 uses, or $0.40 / 20 = $0.02 per guaranteed
use.
Email #2 indicates that the 30-injection syringes sell for $30 per 50 syringes, or $30 / 50 = $0.60 per
syringe. Each syringe is guaranteed for 30 uses, or $0.60 / 30 = $0.02 per guaranteed use. Per
guaranteed use, the two syringes cost the same: $0.02.
Statement 2: Yes. If the customer sticks with the original order, the price for the 1,200 pumps will be
(1,200/10) ($175) = $21,000 less a 5% discount: ($21,000) (0.95) = $19,950. The price for the 50
syringes will be $20 and the price for the 550 needles will be (550/50) ($25) = $275. The total order will
cost $20,245.

If the customer orders an additional 200 needles, she’ll earn a 10% discount on the entire order. Before
the discount, her cost will be $21,000 for the pumps, $20 for the syringes, and (750/50) ($25) = $375 for
the needles, or a total of $21,395. Subtract 10%: ($21,395) (0.9) = $19,255.50. Therefore, the order
that includes the additional 200 needles will be the less expensive order.
There is a neat shortcut for the above math. What is the difference between the two scenarios?
Scenario Scenario 1 Scenario 2 Scenario 2
1 units discount units discount
pumps 1,200 5% 1,200 10%
syringes 50 none 50 10%
needles 550 none 750 10%
The customer would buy the same amount of pumps and syringes under either scenario. For both
products, she receives a larger discount under scenario 2. For the needles, though, while the customer
receives a larger discount under scenario 2, she also has to pay for 200 additional needles. 200 additional
needles will cost an additional (200/50) ($25) = $100. In order for the customer to save money on the
second scenario, then, the 10% discount must save the customer more than $100 on the entire order.
The pumps cost $175 for a box of 10 and the customer wants to order 1,200 of them. 120 boxes at $175
per box is $21,000, so the customer is saving about $2,000 on the pumps alone, compared to half that
(5%, or $1,000) under the first scenario. In other words, the savings on the pumps alone more than
makes up for the extra $100 spent on needles.
Statement 3: Yes. The syringes are guaranteed for a minimum of 20 uses. The needles are “single-use.”
If she orders 50 syringes, they will be able to give a minimum of (50) (20) = 1,000 injections. There are
more than enough syringes, then, to use up the 550 needles she plans to order.

Question 33

Statement 1: Yes. Sort by the “In continuous operation since” column. The 4 breweries that have been in
continuous operation the longest have annual capacities of 1,700, 2,200, 2,200, and 2,700 liters. The
average (arithmetic mean) is the sum of these numbers divided by 4.
1,700 + 2,200 + 2,200 + 2,500 = 8,800
8,800 / 4 = 2,200
The average annual capacity of the other four breweries is the sum of their capacities divided by 4.
(5,800 + 1,000 + 1,500 + 115) / 4 = 2,103.75
The statement is true.
Statement 2: No. Sort by Peating level. The two unpeated whiskies are at the top, with peating levels of
1.5 and 3.5, leading to a median of 2.5. Meanwhile, the other 6 whiskies, which are all peated, are sorted
nicely below. Since the two middle peating levels among those whiskies are both 35, the median is 35. 35
– 2.5 = 32.5, which is NOT more than 35 ppm. The statement is false.
Statement 3: No. The trick to this question is to only pay attention to breweries that have been in
continuous operation for a short time. Ardbeg, Bruichladdich and Kilchoman have been in continuous
operation for the least amount of time. Kilchoman, however, was founded in 2005, so the ratio is 1:1.
That’s out.
Ardbeg was founded 197 years before 2012 (2012 – 1815 = 197), and was in continuous operation for
15 years (2012 – 1997 = 15).
197/15 = 13.13.
Bruichladdich was founded 131 years before 2012 (2012 – 1881 = 131), and was in continuous operation
for 11 years (2012 – 2001 = 11).
131/11 = 11.9.
Ardbeg has a higher ratio, so the statement is false.
(Even if you “add 1 before you’re done” to count the full year of founding and the full year of 2012, you
end up with Ardbeg having a higher ratio.)

Question 34

This short problem is quite difficult because of the subject matter. The first sentence describes a value
that is increasing steadily “by an unknown multiplier” or growth factor. So there is some multiplier that
could be used to calculate the size of the population at the end of every hour. For example, if you know
that a population starts out with 2 bacteria and the multiplier is 5, then at the end of the first hour, there
will be 2 * 5 = 10 bacteria; at the end of the second hour, there will be 10 * 5 = 50 bacteria, and so on.
The second sentence then provides information to calculate that unknown multiplier, or growth factor;
let’s call that multiplier r. At t = 1, the population is p. At t = 5, the population is p2. You are told that the
growth factor corresponds to hourly growth, so there are 4 one-hour intervals in the timeframe
described. So the question becomes this: what value of r, multiplied by itself 4 times (for the 4 one-hour
intervals), would take you from p to p2?
Multiplying r by itself 4 times is the same thing as raising r to the 4th power: (r)(r)(r)(r) = r4. In the four
hours described, the bacteria grew from p to p2 (or p times p) so the bacteria grew by a factor of p.
Therefore, p = r4, or p1/4 = r. (“p to the one-fourth power” is the same thing as the fourth root of p.)
The hourly growth factor, r, is p1/4, or 4√p.
Here’s a way to look at the growth, step by step:
t= given calculation

1 p (given)

2 p × p1/4 = p5/4
3 p5/4 × p1/4 = p6/4 = p3/2
4 p3/2 × p1/4 = p7/4
5 p2 p7/4 × p1/4 = p8/4 = p2
You can now use the growth factor, p1/4, to calculate the population size at t = 0. If you were going from
hour 1 to hour 2, you would multiply the hour 1 population by the growth factor in order to get to the
hour 2 population (as in the above table). Because you are working backwards, from hour 1 to hour 0,
you instead divide hour 1’s population by the growth factor: p / p1/4:
p / p1/4 = p1 – 1/4 = p3/4
“p to the ¾ power” is the fourth root of p3, or 4√(p3).
Column 1: The correct answer is A.
Column 2: The correct answer is D.

Question 35

Statement 1: February 17. Earthquakes of magnitude 3 or greater are coded with darker shades of
gray, all the way to black, whereas magnitude 1 earthquakes are white (on the bottom of the stack) and
magnitude 2 earthquakes are light gray (just on top of the white columns). So the question becomes this:
which day (other than February 14) has the greatest proportion of dark grays & blacks?

By inspecting the columns, February 17 has the greatest number of dark grays & blacks: at least 20,
whereas none of the others has as many. Meanwhile, only February 19 has fewer earthquakes overall, in
fact only slightly fewer (February 17 seems to have 115 earthquakes, while February 19 has at least
111). February 19 only has ~15 earthquakes of magnitude 3 or greater. So the proportion of ~20
magnitude 3+ to 115 earthquakes is the largest of any day other than February 14.

Statement 2: 22%. You can solve this problem by summing up on the calculator all the daily totals,
estimating the exact numbers as best you can: 177 + 133 + 142 + 115 + 115 + 111 = 793. Finally, 177
÷ 793 = 22.3%.

Alternatively, you could estimate more roughly and get to the right answer, since the 5% difference
between successive answer choices is actually rather large. For instance, you could eliminate A quickly by
noticing that 17% is just a shade greater than 1/6 (= approximately 16.7%); however, February 14
showed a markedly greater number than 1/6 of the total. It might be hard to eliminate 27%, but 32%
and especially 37% should seem outlandishly big – after all, 32% is just slightly less than 1/3, and
February 14’s total is not large enough to cover two average days.

Question 36

Statement 1: 274. This problem asks you to find the difference in the number of billionaires between the
US and the total of Russia, the UK, and Germany combined. Because the table is given in percents, you
can first find the total number of billionaires in these two groups. Using the labels on the chart, you can
see that the United States had 44% of the worldwide billionaires, Russia had 6%, the United Kingdom
had 3%, and Germany had 6%. This means that Germany, Russia, and the United Kingdom had 6 + 6 +
3 = 15% of the total worldwide billionaires. You now convert these from percents to actual numbers,
knowing that there were 946 billionaires worldwide.
Number of US Billionaires = (44%)(946) = (0.44)(946) = 416.24 ≈ 416
Number of Russia, UK, and Germany Billionaires = (15%)(946) = (0.15)(946) = 141.9 ≈ 142
Therefore, the difference between these two groups is given by 416 – 142 = 274.
Alternatively, you could subtract the percents first, which would give a 44% - 15% = 29% change. This
also gives 0.29(946) = 274.34 ≈ 274.
Statement 2: 35. You are asked to find the number of billionaires for each country grouped in the
“Other” category, assuming that there are 7 countries in this category and each has the same number of
billionaires. First, you must find the total number of billionaires given by “Other”. Using the table, you see
that “Other” makes up 26% of the total 946 billionaires. Therefore, the total number of billionaires in
“Other” is given by (26%)(946) = (0.26)(946) = 245.96 ≈ 246. Because there are 7 nations contributing
the same number, you divide the total by 7 to achieve the desired result: 246 / 7 = 35.14 ≈ 35.
Alternatively here you could take 1/7th of the percent first, and then calculate that fraction of the total.
This gives 26% / 7 = 3.71%, and so (0.0371)(946) ≈ 35.
(By the way, in actuality, it would be unlikely that there would only be 7 countries in the “Other”
category. Typically, in a pie chart such as this one, the “Other” category would contain countries whose
wedges would all be smaller than that of the smallest country shown individually, which would be Canada
or China at 2%. Since the “Other” wedge is 26%, you’d expect at least 26 / 2 = 13 countries within.
However, you should take the given condition that there are 7 countries in the Other group at face value
and proceed as directed. When you are given a condition, don’t challenge it!)

Question 37

Statement 1: No. Sort by “% wages” to put that column in ascending order. You can readily observe that
the population is also generally increasing from top to bottom, indicating that the correlation between
these two quantities is positive, not negative.
Statement 2: Yes. Here, you need to deal with a derived number: the percent of income derived from
sources other than those listed (wages, salaries, dividends, and interest). If you had Excel and could
build another column, you would add the two percent columns that you do have and subtract them from
100%. Then you would sort by that new column. You want the lowest number in that column, so you
want the highest sum of the two percents. Since the wage numbers seem to have more variability, sort
by that column. Texas is at the bottom, with 76.8% wage income. Together with 4.4% dividends &
interest, you get 81.2% (for 18.8% other income). Will any other states give you a higher sum? As you
scan upwards, you should see that no other state will have as large a sum of % wages and % dividends
& interest (of course, check any possibilities with your calculator). So Texas is the subject of the
sentence.
What about the predicate? Does Texas have an income per capita (meaning per person) of more than
$20,000? Well, divide the income per tax household by the size of tax household: $51,747 / 2.58 ≈
$20,057, which is more than $20,000. The statement is true.
Statement 3: No. To compute the household income in dollars from dividends and interest in any state,
multiply the household income in dollars by the “% dividends & interest” number.
Georgia: $50,320 × 4.5% = $2,264
New Mexico: $44,129 × 4.9% = $2,162
Even though New Mexico has a higher percent of this sort of income, the average income is lower—by
enough to bring the dollar figure below that of Georgia. The statement is false.

Question 38

The charity plans to auction off 100 lots this year. If they average $1,500 per lot, they will raise
$150,000, leaving an additional $100,000 to be raised by pledges. The capacity of the room is 1,098, but
not all of those people will be invited guests; the executive director indicates in the third email that
approximately 10% will be servers and other workers. That leaves 1,098 × 0.9 = 988 invited guests. The
average pledge per guest would need to be $100,000 / 988 = $101, or approximately $100 per guest.
The correct answer is B.

Question 39

Statement 1: No. In the third email, the executive director does introduce the idea of changing to a
larger venue, but says only that “maybe” they “should think about” doing so. The paragraph concludes
with a statement that the current venue should be “fine” if the number projections are accurate,
indicating that the director is willing to stick with the current venue.
Statement 2: Yes. In the second email, the auction coordinator indicates that they raised a total of
$131,000 + $73,000 = $204,000 last year, or $46,000 short of this year’s goal. The coordinator then
indicates that the additional items for auction this year “should go for about the same price on average”
and therefore “should be enough to raise the additional $50,000 needed.” Notice the final 4 words
(emphasis added): the coordinator is inferring that the charity will raise the same amount of money as it
did last year plus the additional $50,000 to meet the new goal.
Statement 3: No. In the second email, the auction coordinator says that the charity “could probably plan
on similar response and attendance rates this year.” This language indicates a degree of confidence, but
not certainty. Note that there is a “mix-up” trap here; later in the same paragraph, the auction
coordinator says “I’m certain we can reach 100 items.”

Question 40

Statement 1: No. For the previous year, 1,378 people attended and there were $131,000 in pledges. The
pledge per person, then, was $131,000 / 1,378 = $95.07 per person. This year, the capacity is 1,098, but
10% of the people in the room will be workers, not invited guests. Therefore, there will be a maximum of
1,098 × 0.9 = 988.2 = 988 invited guests. If they pledge at the same rate, they will pledge a total of
approximately $95 × 988 = $93,860.
There were 58 auction lots last year, from which $73,000 was raised. So the per-item rate was $73,000 /
58 = $1,258 (approximately). This year, the charity plans to have 100 lots; at the same per-item rate,
they can expect to raise $1,258 × 100 = $125,800.
$94,000 and $125,800 add up to less than $250,000.
Statement 2: No. At first glance, this statement seems to ensure the $250,000+ goal: the charity will
have more people attending than last year and will also offer more items for auction than last year. This
statement, however, neglects to provide any information about the pledges made by the guests or the
amount raised by the auction items (either averages or any other figures). We cannot say for sure, then,
how much the charity would raise.
Statement 3: Yes. We can re-use some of our calculations from the first statement. We know that there
are 988 invited guests; if they pledge an average of $150 each, they will pledge $148,200 collectively.
We also know from statement 1 that there were 100 auction lots at an average of $1,258 for a total of
$125,800. That alone is enough to raise the charity above the $250,000 goal, without even accounting
for the higher average ($1,300) given in this statement. This plan indicates that the charity will raise
more than $250,000.

Question 41

Statement 1: strong nuclear. You are told that bosons carry particular forces, and that the blue lines
show what interactions are possible. You are asked to identify the force that affects quarks but not
leptons. Since the bosons correspond to forces, you can rephrase the problem: what boson “touches”
quarks but not leptons via a blue line?

Following the blue lines, you can see that the only boson that touches quarks but not leptons is the
gluon. The notes indicate that the gluon carries the strong nuclear force.

Statement 2: Z boson. Some bosons have a loop back to themselves, indicating that they can interact
with other particles of precisely the same type. Which bosons do not have a loop back to themselves?
Only two: photons and Z bosons, either of which would be possible answers.

Question 42

In the given argument, Colleague A indicates displeasure with one aspect of the current office policy for
“personal event” parties given by employees: because the parties are limited to one per month, everyone
celebrating something is “lumped together” for that month. Note that colleague A does not actually go so
far as to call for individual parties, one for each person celebrating something. Colleague B provides 3
reasons why, in her opinion, offering individual parties would not be a good idea: costs might go up,
productivity might go down, and someone might be overlooked.
We’re asked to find two things. First, we need to find something to which Colleague A would likely object
. The first two statements are “opposites”: there are either “significant disadvantages” (answer A) or
“significant advantages” (answer B) to changing the current policy in this matter. Colleague A does dislike
something about the current policy, so it’s unlikely that this colleague would object to the statement that
there are advantages to changing the policy. Would Colleague A argue that there are no significant
disadvantages to such a policy change? We can’t quite go as far as that – Colleague A mentioned one
advantage but did not discuss disadvantages, nor did he indicate that there are not any disadvantages to
changing the policy.
Answer C says that the company should maintain its current policy; Colleague A is very clear that he
dislikes at least one aspect of the current policy. Colleague A, then, would be likely to object to this
statement. Answers D and E address who should pay for the parties; Colleague A does not address this
aspect of things at all.
Therefore, Colleague A’s objection is that the company should not maintain its current policy. How would
Colleague B be likely to respond? Colleague B listed three problematic consequences of changing to
another policy, so Colleague B would respond that there are significant disadvantages involved with
changing the current policy.
Column 1: The correct answer is C.
Column 2: The correct answer is A.

Question 43

Statement 1: No. Sort the table by Act / Hyp; there are seven sequences in the Hyp category. Scan the
Max Identity category for these seven sequences. Six of them are less than 50%, but Ovalbumin-related
protein Y is 99%.
Statement 2: No. Sort the table by Serpin Clade. The four sequences in serpin clade A all have lengths in
the 400s. The three sequences in serpin clade B all have lengths in the 300s. The average length of
sequences in serpin clade B, then, must be lower.
Statement 3: Yes. Sort the table by Length. 5 of the 11 sequences have a length less than 415; therefore
6 of the 11 have length 415 or greater. If choosing at random from among the 11 sequences, you have a
6/11 chance to choose one with length 415 or greater. This is greater than ½ (or 50%).

Question 44

The question is a classic overlapping sets problem with three groups so we use the Venn Diagram
approach to solve. Construct the Venn Diagram as follows:

The region in the center is the overlap of all three groups, which we know from the problem to be 5. We
are also told that the same number of students overlap between any two groups, which means that the
three overlapping regions in the center (excluding the middle) must all have the same value; we can call
this value x. Since every class must contain 20 students and all the circles are identical, we know that the
same number of students is missing from each class; we can represent this with another variable, y. The
resulting Venn Diagram looks like this:
The question wants us to find a possible number of students who are in only one class (y + y + y = 3y),
and the corresponding number of students who overlap between any pair of classes (5+x). To find out
more information about y and x, we can use the fact that every class must contain 20 students.
Therefore, the following relation must hold:
5 + 2x + y = 20
2x + y = 15
Using the Venn Diagram, we also find that the number of students in only one class is 3y and that the
number of students overlapping between any pair of classes is 5 + x. We proceed by testing the answer
choices in order. Immediately we recognize that the number of students common to any two classes
cannot be the first option, 2, since it must be at least 5.
Next we test the overlap of 7, which means that x = 2. Substituting in we find that
2x + y = 15
2(2) + y = 15
y = 11
This means that the number of students in only one class is equal to 3(11) = 33. Both answers appear in
the table, which means that this is the correct answer. No need to test further choices.
As an alternate solution, we might notice that 2x + y = 15 only has 8 possible (x,y) solutions
because x and y must be integers. Testing systematically we start from x= 1 and find that 5 + x = 5 + 1
= 6. This answer is not in the table, so we rule out x = 1. Trying x = 2 we find that 5 + x = 5 + 2 = 7,
which is in the table, so we proceed. From here we find that y = 13 and 3y = (3)(13) = 39. As this
answer is also in the table, we must have found the correct combination.
Column 1: The correct answer is E.
Column 2: The correct answer is B.

Question 45

Statement 1: 1/2. Looking quickly at the drop down menu choices, we see that we are going to have to
find a fraction comparing the number of households in the Midwest that do not use electricity, to the
number of households in the US that do. This means we will need to find the following:
(Midwest households that do NOT use electricity) ÷ (US households that DO use electricity)
We don’t know these actual numbers in advance, but we do know the total numbers of Midwest and US
households and the percents that use various types of heating sources, so we can calculate.
Let’s start with the number of Midwest households that DO NOT use electricity. First, we need to find the
fraction of Midwest households that fit this qualification. Rather than add up all of the NON-electricity
categories, we can simply take the electricity percent (18%) and subtract that from 100% to get 100% –
18% = 82%. Now, to move from proportion to an actual number of households, we need to find 82% of
the total, which is 25.9 million. Using our calculator, we find that 0.82(25.9) = 21.238 million Midwest
households.
Now we can do the same thing for the number of US households that DO use electricity. First, we can
read from the chart that 35% of US households fit this qualification. Now, to move from proportion to an
actual number of households, we need to find 35% of the total (113.6 million) = 0.35(113.6) = 39.76
million US households.
Finally, we can plug these values into the fraction we are trying to solve: 21.238/39.76 = 0.53 ≈ ½.
Statement 2: 1,662. This question is asking about an average number of households in the Midwest
region per state. That means that we will need to find the total number of households that use either
propane or natural gas (8% and 69% respectively = 77%) and then divide that by the number of states
in the region (12 states). So, to find the total number of states, we need to figure out 77% of the total
(25.9) = 0.77(25.9) = 19.943 million households. To find the state average, we then divide by 12 to get
19.943/12 = 1.6619 million households. But the question asks us for the number in thousands of houses,
so we need to convert millions to thousands. There are 1,000 thousands in a million, so multiply 1.662 by
1,000 thousands per million to get approximately 1,662 thousand. (1.662 million = 1,662 thousand.)
Question 46

This question is exactly like a Critical Reasoning Assumption question. To that end, it may be helpful to
find the basic components of the argument. The conclusion is the last sentence: “Considering this initial
success, DSSCs are going to be the most useful form of solar cell technology.” The premises are, more or
less, everything else.
In order to prove the conclusion, we need to make sure that the other forms of solar cell technologies
(which are not discussed anywhere in the prompt) will be less useful.
A. Possible fact. This statement of fact would provide the most support for our assumption
(F). The two descriptions here (“least expensive and most efficient”) are both superlatives along
important dimensions, expense and efficiency, that would both significantly affect the utility
(usefulness) of DSSCs.
B. Bringing the cost down is positive, but it’s not enough to make DSSCs the MOST useful form
of solar cell technology. We don’t know, for instance, how those cost decreases compare to those
possible with other technologies.
C. Again, this is good news for DSSCs, but the “plausibility” of DSSCs does not make them the
best solar cell technology available.
D. The efficiency of the nanoparticles is irrelevant.
E. It’s nice that this efficiency is “above average”; however, it provides at best only weak
support for the assumption, since we need to assume that DSSCs are the most useful.
F. Required Assumption. This is the assumption we need. If no other forms of solar cell
technologies will be more useful than DSSCs, than DSSCs will have to be the most useful.
Column 1: The correct answer is F.
Column 2: The correct answer is A.

Question 47

Statement 1: Slovenia. The average national fourth grade TIMSS scores for 2003 and 2007 are coded in
red and green respectively. This question asks us to find a nation that has a lower red and a higher green
TIMSS score than New Zealand.
New Zealand’s TIMSS scores fall below more than half the countries on the graph, so it is advantageous
to start by finding the countries that have a lower red score, as fewer options will remain. Of the
countries shown, only Iran, Norway, and Slovenia meet this criterion. We now search among the
remaining three countries to find one that also has a higher green TIMSS score. Of the three, only
Slovenia matches.
Statement 2: Hong Kong, China. You can solve this problem by calculating the percent change:
(Final – Initial) / Initial
Calculate this percent change from 1995 to 2007 for each of the five options in the dropdown list.
Estimating by eye and using the calculator we find that:
Australia: (515 – 495) / 495 = 4.04%
Hong Kong, China: (605 – 555) / 555 = 9.09%
Iran, Islamic Republic: (400 – 380) / 380 = 5.26%
Norway: (470 – 475) / 475 = -1.05%
United States: (525 – 515) / 515 = 1.94%
Note that it is much easier to calculate using only the 5 answer choices than to calculate using all
countries on the graph.
Question 48

To solve this Rate-Time-Distance problem, we first recognize that Car A starts off behind Car B but then
gains enough to actually be ahead in 4 hours. This means that we have a situation where Car A
is chasing Car B, telling us that the two cars are working against each other (Car B keeps driving, making
it harder for Car A to actually catch-up). When things work against one another, we can simplify by
subtracting their rates to get the relative rate at which Car A is gaining on Car B. Let A denote the speed
of Car A and B denote the speed of Car B, so their relative rate is given by A – B. Physically, this is the
rate at which Car A is gaining on Car B.
We are then told that Car A starts out 20 miles behind Car B but finishes 4 miles in front at a time 4
hours later. Thus Car A gains on Car B by a total of 20+4 = 24 miles in 4 hours. We now make use of the
formula Rate × Time = Distance. Notice that the combined rate, A – B, multiplied by the time of 4 hours
must give us a total distance of 24 miles. Solving for A – B we find that:
(A – B) × 4 = 24
A – B = 6.
Therefore, the speed of Car A must be 6 mph more than the speed of Car B, so we look for answers in
the table that can match this criteria. The only two values with a difference of 6 are 54 mph and 48 mph.
As Car A is the faster car, we mark its speed as 54 mph and Car B’s speed as 48 mph.
We could also solve this problem algebraically. Car B has a rate of B, travels for 4 hours and will go some
unknown distance d. Car A has a rate of A, also travels for 4 hours, but will have to travel not
only d miles but also the 20 miles it is behind at the start, as well as the 4 miles it goes beyond B. This
gives us 2 equations:
4B = d
4A = d + 20 +4
Substituting in for d to gives the following relationship for A and B:
4A = 4B + 24
4A - 4B = 24
4(A – B) = 24
A–B=6
Column 1: The correct answer is D.
Column 2: The correct answer is B.

Question 49

Statement 1: 50. If the five smallest states are estimated (under the given simplification) to have the
same population each, then they have about 600,000 people each (according to the text), or 3,000,000
altogether. From there, we have to approximate the number of farmers markets.

The five smallest dots are all to the left on the graph. Two have about 35 farmers markets each; one has
about 25 markets; one has about 10; and the last has about 55. Together, that adds up to 160 markets
for 3,000,000 people. To change the denominator to 100,000 people, we divide by 30, so we must do the
same to the top, yielding 5.33…, or approximately 5 farmers markets per 100,000 people. Notice that if
you misidentify one of the states (say, picking one of the cluster around 70-80 markets each), it will not
make a real difference, since the answer choices all differ by powers of ten.
Statement 2: 1 : 13,000. The two largest states are clearly the two on the right. One looks slightly
bigger than the other, so we can estimate their populations at 13,000,000 and 12,000,000, for a
combined total of 25,000,000. From there, we need to estimate the number of obese people in each
state. The 13,000,000 state looks to be at about 28.75% (halfway between 27.5 and 30), while the
12,000,000 state is slightly higher, more like 29.5%.
13,000,000 × 0.2875 + 12,000,000 × 0.295 = 3,737,500 + 3,540,000 = 7,277,500, which is the number
of obese people in the two largest states. Now, we simply find the number of farmers markets in those
two states, which looks to be about 300 + 275, or 575.
7,277,400 ÷ 575 ≈ 13,000, or 13,000 : 1. That’s the ratio we’re looking for.

Question 50

Statement 1: 2005. The table is already sorted by year, so there is no need to re-sort to answer this
question. Of the 9 movies that were filmed in 2004, 4 had English as one of the spoken languages. 4/9 ≈
44.4%. Of the 12 movies that were filmed in 2005, 6 had English as one of the spoken languages. 6/12 =
50%. A greater percentage of movies in 2005 had English as one of the spoken languages.
Statement 2: 2004. Sort by the “B/W?” column, and look for the “No” entries. The movies that were shot
entirely in color in 2004 had lengths in minutes of 106, 105, 85, 90, 88, and 76. The movies that were
shot entirely in color in 2005 had lengths in minutes of 100, 80, 75, 96, 91, 92, 80, 95, and 90.
The average length of the movies from 2004 = (106 + 105 + 85 + 90 + 88 + 76) / 6 = 91.7 minutes.
The average length of the movies from 2005 = (100 + 80 + 75 + 96 + 91 + 92 + 80 + 95 + 90) / 9 =
88.8 minutes.
Statement 3: 2005. Sort by the “Medium” column. 6 movies were filmed in 2004 using 35mm film. Of
those, only 1 was also shot in black and white. There were also 6 films shot in 2005 using 35mm film. Of
those, 2 were also shot in black and white. A higher percentage of films shot in 35mm film in 2005 were
also shot in black and white.

Question 51

Statement 1: No. The information to answer this statement is all contained in the third tab. The last four
elements have the shortest half lives, and only three of the four allow EC as a decay mode (element
#100, fermium, does not).
Statement 2: Yes. First, check the possible decay modes of californium. They are listed as Alpha particle
emission and Spontaneous Fission (SF). Now, we must turn back to the descriptions of the various decay
modes provided in the first tab. SF has no effect on the mass number of an element, which leaves only
Alpha particle emission. This decay mode is described as causing an element to drop “the atomic number
by 2 and the mass number by 4.” The atomic number of californium is 2 greater than the atomic number
of curium, and the mass number is 4 greater. This fits the description of the decay mode exactly.
Statement 3: Yes. This statement revolves around the percent difference formula (difference/”original” *
100), and a basic understanding of statistics. Because there are 11 elements listed in this question, the
median will typically be the 6th element (counting from either the top or the bottom). All the relevant data
for this statement is to be found in the second tab. However, because there are only six data points for
density, the median density will actually be the average of the melting points of the third and fourth
elements.
The element with the median melting point is Californium, at 1,173 degrees. The highest melting point is
1,900 degrees. The percent difference is (1,900 – 1,173) / 1,900 =0.3826, which gives us a percent
difference of 38.3%. You are told that the “original” (what you’re taking the percent of) is the highest
melting point, so that’s why we divide by 1,900.
The median for density is the average of the middle two densities, 15,100 and 14,780. (15,100 + 14,780)
/ 2 = 14,940. The highest density is 20,450. The percent difference will thus be (20,450 – 14,940) /
20,450. Just by looking, we can see that this will be about 25%, which is much smaller than the other
percent difference. (The calculator tells you 0.269…)

Question 52

If an element has more than one possible decay mode, it is impossible to know its atomic number after
one cycle of decay (because three of the four decay modes change the atomic number in three different
ways, and the fourth, spontaneous fission or SF, breaks the nucleus apart entirely, changing the atomic
number unpredictably). So, to know the result, we’d need to be sure that the element had only one
decay mode (and that decay mode can’t be SF).
Statement 1: Cannot Be Determined. Neptunium (#93) has two decay modes.
Statement 2: Can Be Determined. Mendelevium (#101) has only one decay mode, which is not SF.
Statement 3: Cannot Be Determined. Einsteinium (#99) has three decay modes.

Question 53

To begin, we must isolate the elements that meet the initial criterion, ‘two possible decay modes.’
Looking at the information in the third tab, we see that six elements have two possible decay modes: 93,
94, 95, 98, 100, 103.
In order to answer the question, we need the statistics from the second tab for the six elements with two
decay modes:
Mass # of
Smallest Mass #
Atomic Most Stable
Element of an
Number Isotope
Attested Isotope
(MSI)
93 Neptunium 237 225
94 Plutonium 244 228
95 Americium 243 231
98 Californium 251 237
100 Fermium 257 242
103 Lawrencium 262 251
At this point, the description given in the first tab comes in handy. “An element is defined by its atomic
number, the number of protons in the atomic nucleus...The mass number of an isotope is the total
number of protons and neutrons.” This tells us that the number of neutrons in a given isotope is simply:
mass number minus atomic number. But the atomic number is the same for every isotope of a given
element! That means that the difference between the mass number of the most stable isotope and the
smallest mass number of an attested isotope IS the difference between the neutrons (as the proton
number always remains the same).
From here, finding the answer is relatively straightforward. We just subtract the value in the last column
from the value in the second to last column for each element (we can skip Californium, as it doesn’t show
up in the answer choices). The largest difference is 16, for Plutonium. Plutonium has an atomic number
of 94, which is also its number of protons.

Question 54

The only way to answer this question is to quickly work out all the possible patterns of 6 rounds that
would result in a score of 4 points. Basically, Player X could have won 1, 2, or 3 games (any more than
that and the score is too big, while draws alone can’t bring the score up to 4 points in only 6 games).
Let’s consider these three possibilities:
1 Win: With only 1 win, the only way to get up to 4 points would be to draw the next 4 games, and sit
the last one out. This will turn out to be the situation that the answer choices fit.
2 Wins: Looking at the answer choices, we can see that 2 isn’t an option. However, with 2 wins, the only
way to reach 4 points is to draw twice and then lose once (and sit the final round out).
3 Wins: With three wins (adding up to six points), the only way to get down to 4 points is to lose two
matches in a row (and sit out the final round). While 3 is an option in the answer choices, we already
know that 2 isn’t, so this can’t be the correct situation.

Question 55

Since the “Butterflies of Asia” exhibit already has two Asian butterflies and one non-Asian butterfly (the
Australian Painted Lady), any Asian butterfly can be added to it; however, no butterfly that is not found
in Asia can be added.
Similarly, since the “Blue Butterflies” exhibit already has two blue colored butterflies and one butterfly
that is not either blue or purple (the Great Nawab), any blue or purple butterfly can be added to it, but
no butterfly that is not at least partially blue or purple can be.
So a butterfly that CAN be added to both groups must be blue and Asian. A butterfly that CANNOT be
added to either group must be neither blue nor Asian.
Option F – Could be added to either exhibit
The Emerald Swallowtail butterfly is both partially blue and lives in South Asia, so it could be added to
either exhibit.
Option D – Could NOT be added to either exhibit
The Monarch Butterfly contains no blue and its natural habitat is only North America, so it can’t be added
to either exhibit.
Options A, B, C, and E
All of these butterflies are either found in Asia or partially blue or purple, but not both, and so could be
added to one exhibit but not the other.
Column 1: The correct answer is F.
Column 2: The correct answer is D.

Question 56

The question is this: what number P must be in the pool, and what number T must be on the team, such
that when you choose T from P, you have between 20 and 25 possibilities (not-inclusive)? We also
need T to be at least half of P.

This is a combinatorics problem. The formula we want to use for the selection of a team (for which order
of selection doesn’t matter, only the composition of the team) is this:
The “Ins” represent the team T, and the “Outs” represent those not chosen, or P – T.

To get a number as high as 21 to 24, we should start at the high end for the pool P, and then test
possibilities for T.

8!/(8!0!) = 1 (always just 1 way to choose all the candidates from a pool!)
8!/(7!1!) = 8 no
8!/(6!2!) = (8)(7)/2 = 28 no
8!/(5!3!) = (8)(7)(6)/6 = 56 no
8!/(4!4!) = (8)(7)(6)(5)/24 = too big
Notice that you don’t have to check T = 3 or lower, because T must be at least half of P.

Now try P = 7, skipping right to T = 6 and going down.

7!/(6!1!) = 7 no
7!/(5!2!) = (7)(6)/2 = 21 yes!

There can be only one set of answers, so we can stop here.


Column 1: The correct answer is E.
Column 2: The correct answer is C.

Question 57

Statement 1: 1/3. In order to solve this problem, you need to remember that square footage is a
measure of area, and area = length × width.
By inspecting the diagram, you can see that the width of the apartment is the width of the two bedrooms
taken together, or 19.5 feet. You can also see that the length of the apartment is 41 feet and the length
of the two bedrooms is 13.5 feet.
So the area of the two bedrooms taken together is 19.5 × 13.5, and the area of the entire apartment is
41 × 13.5. So the square footage of the bedrooms is (19.5 × 13.5)/(41 × 13.5) of the area of the
apartment. This simplifies down to 19.5/41 = 0.3293, which is much closer to 1/3 = 0.3333 than to ¼ =
0.25, 3/8 = 0.375, 2/5 = 0.4, or ½ = 0.5.
Statement 2: 45 feet. In order to solve this problem, you need to recognize that the apartment is a
rectangle (you are told to ignore the outdoor patio). The distance from one corner of the apartment to
the farthest corner from it is the length of the diagonal of the rectangular apartment.
From the diagram, you can determine that length and width of the apartment are 41 feet and 19.5 feet,
respectively. The Pythagorean theorem, which is a2 + b2 = c2, can then be used to calculate the length
of the diagonal of the apartment.
So the length of the diagonal is the square root of 19.52 + 412. Use the IR calculator to compute these
messy numbers! 380.25 + 1681 = 2061.25, and the square root of 2061.25 can be calculated using the
IR calculator’s square root key (sqrt). Doing so, you get 45.4010. The closest answer is 45 feet.

Question 58

Option B - strengthen
Most of the growth in the new suburbs is on the north side of the city, and it is expected that most of the
new residents will work at the large companies headquartered in that area, primarily using Austia’s
freeways only to travel to central Austia during off-peak hours.
This option tells us that the new residents will not use Austia’s freeways during high traffic commute
times as much as the existing residents do because they will not be leaving the area to get to work.
Option C - weaken
Since Austia has limited public transportation outside of the city core, most commuters who drive from
the outlying neighborhoods to the city center have no practical alternative to driving available.
This option tells us that the new residents cannot use public transit instead of driving to the city center
will have to use Austia’s freeways to drive to the city center to work, adding to the freeway congestion.
Options A, D, E, and F
These options do not directly address the issue of the new population’s freeway utilization, and so do not
significantly strengthen or weaken the conclusion.
Column 1: The correct answer is B.
Column 2: The correct answer is C.

Question 59

Statement 1: Yes. Sort by DWT. There are only 3 TC vessels capable of transporting over 50,000 metric
tons: Amore Mio II, Ayrton II, and Agamemnon II. For each, we need to multiply the daily rate (net) by
30.
Amore Mio II: 30 × 25,000 = 750,000
Ayrton II: 30 × 22,000 = 660,000
Agamemnon II: 30 × 22,000 = 660,000
In all three cases, the charterer is paying less than $800,000 per month to the owner.
Statement 2: Yes. Sort the table by Type. There are 8 BC vessels and 11 TC vessels. The median of the
BC vessels will be between the 4th and 5th. Count from lowest to highest to find the 4th and 5th, which are
both $15,000. This means the median of the BC vessels is $15,000.
Because there are 11 TC vessels, the median will be the 6th highest or lowest value. The 6th is only
$12,838.
Statement 3: Yes. Sort by Class. Though many ships can be categorized as Ice Class IA, we can ignore
anything in the BC category, because the daily operating expenses are so low. This leaves 8 ships. From
here, we can quickly eyeball our way to the ship with the highest Daily operating expense to Daily rate
ratio: Alkiviadis. 7,000/12,838 = 54.5%, which is less than 55%. You can use 50% as a benchmark,
noting that all the other ships in the Ice Class IA category have daily operating expenses that are less
than half of the daily charter rate.

Question 60

Statement 1: 130. FHR (fetal heart rate) is tracked on the top graph. You are told that baseline FHR is
the average FHR over a 10-minute window, excluding certain time periods. One of the excluded periods
is a deceleration, and you are told that between minutes 3 and 5, the ultrasound shows a deceleration.
So you must exclude all the FHR data between minutes 3 and 5 (the long dip and recovery). The rest of
the heart rate data is clearly centered on 130 bpm, with only slight variability.

Statement 2: 70. In the last sentence of the note, you are told that “the duration of a uterine contraction
is measured from the onset of a sharp increase in uterine pressure from a baseline (typically 0 mmHg as
shown) to the return to the baseline.” The lower graph demonstrates just such a sharp uptick in pressure
starting at 120 seconds (the start of minute 2). This elevated pressure lasts until 190 seconds (one tick
past minute 3). 190 – 120 = 70.

Question 61

Statement 1: B - Low-fat milk (1%) and skim milk. You are looking for the
greatest magnitude percent change, so a large percent decrease could be the winner. Let’s investigate
each case by comparing visually the size of the column segment in 2005 with the corresponding segment
in 1980, looking for large proportional changes either up or down.
a) Flavored milks: Very little proportional change in size from 1980 to 2005.
b) Low-fat milk (1%) and skim milk: Appears to almost double in size, which would represent nearly a
100% increase.
c) Plain reduced-fat milk (2%): Very slight growth, proportionally.
d) Plain whole milk: A big decrease in absolute terms, but percent-wise, only a little more than a 50%
decline – certainly not close to 100% decline.
So the clear winner in percent terms is low-fat, with its nearly 100% increase. Closer inspection would
reveal an approximately 80% increase, whereas plain whole fell only about 60%.
Statement 2: A - 1980. This answer should jump out: the absolute amount of plain reduced-fat milk
consumed was lowest in 1980, by visual inspection. In that same year you had the highest overall milk
consumption. So it must be that the ratio of (a) plain reduced-fat to (b) overall milk consumption, or
quantity (a) divided by quantity (b), was lowest in 1980.

Question 62

The passage as a whole strongly suggests that foreshock, main shock, and aftershock are part of the
same causal chain of events because the foreshocks, main shock, and aftershocks all occur on the same
system of faults and are temporally related.
Option A - All main shocks are preceded by foreshocks.
Although we do know that at least some main shocks are preceded by foreshocks because the third
sentence, which says that a foreshock is only a foreshock if it occurs before a larger quake on the same
fault system, we do are not told whether all main shocks are preceded by foreshocks.
Option B - All main shocks are preceded by foreshocks.
Although we do know that at least some main shocks are followed by aftershocks because of sentence
three, which says that an aftershock is only an aftershock if it occurs after a larger quake on the same
fault system, we do are not told whether all main shocks are followed by aftershocks.
Option C - Aftershocks are more common that foreshocks.
The passage doesn’t give us any information about the frequency of aftershocks relative to foreshocks.
Option D - Foreshocks are generally weaker than aftershocks.
Although the passage says that neither foreshocks nor aftershocks are as strong as main shocks, it does
not give us any information about their strength relative to each other.
Option E – Main shocks can be triggered by foreshocks. - TRUE
Since a foreshock is a smaller earthquake that happens before the bigger main shock on the same fault
system, and there is a probability that a small earthquake can trigger a larger earthquake on the same
fault system that depends on the smaller earthquakes “location and interaction with the fault system,” it
must be possible for some foreshocks to trigger main shocks.
Option F – An aftershock can be a bigger earthquake than the main shock that preceded it. -
FALSE
The fourth sentence says that an aftershock occurs only after a bigger quake on the same fault system,
which is the exact opposite of what this statement says so it cannot be inferred from the passage.
Column 1: The correct answer is E.
Column 2: The correct answer is F.

Question 63

Statement 1: D - $9,765,600,000. This graph does not give the exact per person spending of
Minnesota and Iowa. Instead, it gives ranges of possible values. To find the greatest possible difference
between the total spending of Minnesota and Iowa, you need to assume that Minnesota spent
the maximum amount per person ($2,712) in its category (darkest). Meanwhile, assume also that Iowa
spent the minimum amount per person ($1,536) in its category (one step up from the lightest/bottom).
Multiply those values by the populations of the respective states to calculate total public and elementary
school expenditures for each state:
Minnesota = 5.3 million × $2,712 = $14,373,600,000
Iowa = 3 million × $1,536 = $4,608,000,000
Now subtract to find the difference:
$14,373,600,000 − $4,608,000,000 = $9,765,600,000
Statement 2: B - 3.8 million. The easiest way to start answering this question is to first find a range for
the possible values of the total state expenditures of Missouri. Multiply Missouri’s population by both the
maximum possible expenditure per person and by the minimum possible expenditure per person to find
the range:
Min = 5.9 million × $1,161 = $6,849,900,000
Max = 5.9 million × $1,535 = $9,056,500,000
This information gives us the range of Kansas’ total expenditures as well, because we know that the two
states spent the same amount. Now we need to use these values to find the range of possible values for
the population of Kansas.
The formula for population that we can use is this:
Population = Total Expenditures ÷ Expenditures Per Person
To verify this conceptually, imagine a state that spent $1,000,000,000 on Total Expenditures, and
imagine that you separately know that the state spent $1,000 per person. Then there must be 1,000,000
people in that state.
Now, to find the minimum possible population of Kansas, we need to divide the smallest possible Total
Expenditures by the largest possible Expenditures Per Person.
Minimum Population = $6,849,900,000 ÷ $1,970 ≈ 3.5 million
Similarly, to find the maximum possible population, we need to divide the largest possible Total
Expenditures by the smallest possible Expenditures Per Person.
Maximum Population = $9,056,500,000 ÷ $1,843 ≈ 4.9 million
The only answer in that range is 3.8 million.

Question 64

Statement 1: Yes. Sort by the Washer and Dryer column. Note that all three of the units with a washer
and dryer also have a pool.
Statement 2: No. Sort by the Square Feet column. Since there are 10 (an even number of) apartments
on the list, the median square footage is the average of the square footage of the 5th and 6th (middle
two) apartments, or 660 and 678. Since the average (arithmetic mean) apartment size is given as 692
square feet, and we know that the median must be less than 678, this statement cannot be true.
Statement 3: Yes. In order to solve this problem, you need to know that a positive correlation between
rent per square foot and the state of having either covered or garage parking means that apartments
with higher rents per square foot are more likely to have either covered or garage parking.
You can see that this is the case by sorting by rent per square foot. Note that the four apartments with
the highest rents per square foot also have either covered or garage parking and that none of the
apartments with lower rents per square foot have either covered or garage parking.

Question 65

To solve this statistics question, we must first recall the Average Formula, which states that the
Average = (Sum) / (Number of Terms).
Rearranging, we see that
Sum = (Average) × (Number of Terms).
Therefore, when the prompt tells us that the average of all 4 numbers is 18, it’s really telling us that the
sum of all four numbers is equal to (18) × (4) = 72. As we know that one of the large marble was 48, we
know that the sum of the 3 remaining numbers is given by 72 – 48 = 24. Therefore, the sum of the other
two large marbles and the small marble must be 24. This tells us to look among the answer choices for
pairs that add up to 24.
Within the table there are two such pairs: (11, 13) and (5,19). Because the small marble is limited to the
range 1-10, inclusive, and both numbers in the set (11, 13) are greater than 10, neither of these could be
the value of the small marble. Therefore, we are left with just one set: (5,19). As 5 is less than 10, we
choose this value for the small marble and leave 19 as the sum of the other two large values.

Question 66

To solve this logic problem, we place the parents and children in their respective groups working from the
most to the least restrictive clues. The most restrictive clue states that X is in group 2:
__________ _X________ __________
From here, we know that A and B must be split among groups 1 and 3. This means that one place in
groups 1 and 3 are occupied by A and B, leaving only one “child” space in each of these groups. As no
group can have more than two children and E must be in the same group as F, this means that both E
and F must be in group 2.
__________ _X_E_F____ __________
Another clue tells us that if F is in group 2 then A must be in group 1.
__A_______ _X_E_F____ __________
Consequently, since A and B must be split among groups 1 and 3, we know that B must be in group 3.
__A_______ _X_E_F____ __B_______
Finally, the only clue we have no used is that Y must be in a different group that A and D, meaning Y
cannot be in group 1.
Scanning the answer choices, we see that there are three adults that could potentially be in group 1 – X,
Y, and Z. From the above we have ruled out X and Y, and so only Z could still be an adult in group 1. The
possibilities for children in group 3 are A, D, and E. From the above, we know that neither A nor E can be
in group 3, so the only possibility is D.
Column 1: The correct answer is the 6th choice.
Column 2: The correct answer is the 2nd choice.

Question 67
Statement 1: Yes. In tab 2, we learned that the doctoral candidate proposed a study on humans. An age
range for the study subjects was not specified, so both committees 1 and 2 would have had to accept the
proposal in order for this study to be approved.
Statement 2: Yes. In tab 2, the botany professor’s proposal targets “all living organisms native to a
nature preserve.” Committees 1 through 5 all deal with living organisms, but the nature preserve “usually
does not permit access to people,” so no people are living in the preserve. The relevant committees,
then, must be committees 3, 4, and 5. Committees 3 and 4 must have a minimum of 4 members and can
pass a proposal with at most a single “no” vote, so the botany professor must have received a minimum
of 6 “yes” votes from these two committees combined. Finally, committee 5 must have at least 5 people
and a simple majority is needed to approve a proposal, so the professor must have received at least 3
“yes” votes from this committee, for a total of at least 9 “yes” votes.
Statement 3: No. The first paragraph of the first tab told us that the task of the committees is to “ensure
that the proposed research complies with all laws and meets the university’s standards for ethical
experimentation.” If a committee rejected a particular proposal, then that proposal must have failed to
meet at least one of these two criteria, but it is entirely possible that a proposal might pass an ethical test
and yet still fail to comply with some law.

Question 68

To compute the minimum number of board members, we must determine how many people we need to
staff each of the 6 committees at the minimum level. The second paragraph tells us that each committee
must have a minimum of 4 members; there are 6 committees, so it is tempting to say that we must need
a minimum of 24 board members. Each board member, however, is allowed to sit on up to 3 committees.
In addition, committees 5 and 6 must have an odd number of members – and 4 is not an odd number.
Let’s begin by assuming that, if we want to minimize the number of total board members, then we must
allow each member to sit on the maximum 3 committees. Further, because there are 6 committees total,
let’s group our board members into two large groups. The first group will sit on committees 1, 2, and 3.
How many members do we need to fill those three committees? These three committees can have the
minimum 4 members each, so we need 4 board members total in order to fully staff the first three
committees.
What about the second set of three committees? Committee 4 can have a minimum of 4 members, but
committees 5 and 6 must have an odd number of members (and a minimum of 4). Committees 5 and 6,
then, must each have a minimum of 5 members. The same 5 people can sit on both committees, and 4
of those 5 can also sit on the 4th committee, so we need a minimum of 5 board members to fully staff
committees 4, 5, and 6.
Together, we need a minimum of 4 + 5 = 9 board members to fully staff all six committees.
The correct answer is C.

Question 69

Statement 1: No. It is true that the undergraduate’s proposal had to be reviewed by both committees 1
and 2 (because no age range was specified for the human subjects of the study). It is also true that each
individual committee must vote unanimously in favor of a proposal in order for it to be accepted. But it is
possible that only one of the two committees rejected the undergraduate student’s proposal. It is
possible, therefore, that only one person on one committee rejected this proposal.
Statement 2: No. Under such circumstances, her proposal might be accepted, but it does not absolutely
have to be true that it will be accepted. Tab 2 indicates that the sociology professor’s proposal was
rejected when a certain fact became apparent, but does not indicate whether there may have been
additional factors that would also have resulted in rejection. It might be the case that, even if she fixed
the one noted problem, another problem may exist or arise. The way in which she fixes the existing
problem might itself be something that is cause for rejection.
Statement 3: Yes. A study involving humans would require the approval of Committees 1 and 2. The pets
would require the involvement of committees 3 and 4 (mammals and all other animals). All four
committees must have a minimum of 4 members, and the researcher would have to gain unanimous
approval from Committees 1 and 2 (4 votes each) and at least 3 “yes” votes each from Committees 3 and
4, for a minimum of 14 votes.

Question 70

We need to maximize the area of a cylinder by selecting a radius and a height given certain limiting
parameters. First, examine the formula for the volume of a cylinder:V = πr2h. The value for π is a
constant – that is, it will never change – so we can ignore it for our purposes.
We can control two variables, therefore: the radius and the height. In the formula, the radius will be
squared, so to maximize the area, we should make the radius as large as possible. Once we do that, the
height will be determined, because the radius and the height must add up, at most, to 16.
We’re told that “the height of the cylinder cannot be more than twice the radius, nor can the reverse be
true.” The second half of that sentence is the most important: the radius cannot be more than twice the
height. If the two add up to 16, then what would the radius and height have to be in order for the radius
to be exactly twice the height?
Divide 16 by 3. The height would be 5 1/3 and the radius would be twice that, 10 2/3. The problem
specified, though, that the values must be integers. Should we set the radius at 10 or 11? 11 is the
bigger number… but then the height would have to be 5 and the radius would be more than twice as big
as the height. 10, then, is the largest possible value for the radius, and if the radius is 10, then the
maximum value for the height is 6.
Column 1: The correct answer is D.
Column 2: The correct answer is B.

Question 71

Statement 1: No. First we should verify that Fairview had the lowest profits during the second quarter
(sort by Q2 Profits). We see that they do, in fact, have the lowest profits, so now we must test the
second part of the statement: “Fairview has the lowest profit margin during the same quarter.” To find
the profit margin for Fairview, we will use the formula Profit Margin = Profits / Revenues.
During the second quarter (Q2), Fairview had profits of $1,130,000 and revenues of $5,014,000.
Thus, Fairview’s profit margin that quarter was $1,130,000/$5,014,000 ≈ 0.23, or 23%.
Now, to determine whether Fairview has the lowest profit margin we could try to calculate this value for
every city, but that would take far too much time. It would be better to try to estimate values and then
test those that might seem likely. We know that Fairview has the lowest profits, so every other city will
have a larger numerator (potentially making Profit Margin larger), but if we can find a city whose profits
are similar but whose revenue is substantially greater, that would be worth testing.
Let’s look at the list of cities around Fairview in this sort, e.g. Mt Pleasant, Greenwood and Lexington. All
of these cities have similar profits to one another. Of these cities, Greenwood has by far the largest
revenue, a value almost double that of Fairview. This makes Greenwood a likely candidate to have a
smaller Profit Margin; let’s test.
Greenwood’s profit margin = $1,627,000/$9,982,000 ≈ 0.16, or 16%.
We have found a city that has a smaller profit margin than Fairview does, so Fairview’s cannot be the
lowest.
Statement 2: Yes. To determine the cities with the fewest stores, we must sort by column Stores. From
this list we see that the four stores with the fewest are York (4), Milton (5), Oak Grove (5), and
Springfield (6). Now we need to scan across the rows to see what happened to profits across the 4
quarters for each city, and determine if only one city saw profits fall steadily across all 4 quarters. We
must be careful not to only look at the difference between Q1 and Q4, because an overall decrease is not
the same as a steady decrease (meaning a decrease from Q1 to Q2, then another decrease from Q2 to
Q3, and then a final decrease from Q3 to Q4).
York saw a decrease from Q1 to Q2 (from $3,908 to $1,945, in thousands), and from Q2 to Q3 (from
$1,945 to $1,631), but an increase from Q3 to Q4 (from $1,631 to $2,169). Although York did see an
overall decrease in sales from Q1 to Q4 (from $3,908 to $2,169), it did not see decreases between all
quarters, so it did not have a steady decrease across the entire period.
Milton saw an increase from Q1 to Q2 (from $3,879 to $11,474), so cannot have a decrease in all
quarters (incidentally, Milton saw an increase over the entire period, so Milton could not have had a
steady decrease across all periods).
Oak Grove saw a decrease from Q1 to Q2 (from $2,848 to $2,094), from Q2 to Q3 (from $2,094 to
$1,017), and from Q3 to Q4 (from $1,017 to $731), so Oak Grove saw a decrease across all quarters.
Springfield saw a decrease from Q1 to Q2 (from $5,809 to $3,851) but an increase from Q2 to Q3 (from
$3,851 to $8,834). Although Springfield did see an overall decrease in sales from Q1 to Q4 (from $5,809
to $970), it did not see decreases between all quarters, so it did not have a steady decrease across the
entire period.
Therefore, only Oak Grove saw a steady decrease in profits across all four quarters.
Statement 3: Yes. We want to try to find the city with the largest positive change in revenues from the
first to the fourth quarter = Q4 Revenue – Q1 Revenue. Since we are trying to make this number as large
as possible, it is easier to sort by the first number (as this moves in the same direction as our goal
number). Therefore we sort by column Q4 Revenue and look for the largest value. Portsmouth has Q4
Revenues of $83,796 (in thousands) and Q1 revenues of $20,681, for a difference of $63,115. This is a
very large change, but is it the largest? We could calculate the value for every city, but notice that no
other city has Q4 Revenues above $50,000. This means that they could NOT have increased revenues by
more than their Q4 values, so $63,115 must be the largest dollar increase in revenues between the two
quarters.
Now we just need to determine if Portsmouth had an increase in profits that exceeded $25,000,000.
Remember, the numbers shown are in thousands of dollars, so add three zeroes.
Q4 Profits – Q1 Profits = $35,452,000 - $6,004,000 = $29,448,000 > $25,000,000.

Question 72

Statement 1: D - 220%. First, be sure to read the dotted line (2000) on the left side (male). Next, be
careful about the vertical divisions—the minor gridlines represent 4 (hundred thousand), because they
separate a division of 20 into 5 smaller parts.
Reading carefully, you can see that the population of 35-to-39-year-old men was about 113 (hundred
thousand), whereas the population of 60-to-64-year-old men was about 52 (hundred thousand). Now, as
a percent of 52, 113 is 113/52 × 100% ≈ 2.17 = 217%. The closest answer is 220%.
Statement 2: A - 40%. Again, read carefully! The population of 50-to-59-year-old women was about 70
(hundred thousand) in 2000, whereas it was about 97 (hundred thousand) in 2009. The percent change
is then (97 – 70)/70 × 100% ≈ 0.39 = 39%. The closest answer is 40%.

You might also like